Download as pdf or txt
Download as pdf or txt
You are on page 1of 115

LONG EXAMINATION -1

(2021 - 2022)
SURE
UNSURE

DR. NACIS, DR. LOPEZ

1. What is the minimum rise of BHcg you expect in 48 hours from an early progressing intrauterine
pregnancy?
A. 67%
B. 12%
C. 23%
D. 53%

2. Which of the following factors is least likely to be linked with higher first trimester miscarriage
rates?
A. Diabetes mellitus
B. Parvovirus Infection
C. Maternal age older than 40 years
D. Obesity

3. Which of the following is the most common cause of Consumptive Coagulopathy?


A. Dead fetus
B. Ectopic Pregnancy
C. Retained Products of Conception
D. Abruptio Placenta

4. FOR FISSURE NIPPLES, WHICH OF THE FOLLOWING IS NOT RECOMMENDED TREATMENT OR


REMEDY?
A. USE NIPPLE SHIELD TO PROTECT THE AFFECTED BREAST
B. WASH THE BREAST WITH WATER AND MILD SOAP
C. ALL OF THE CHOICES
D. CONTINUE BREASTFEEDING ON THE AFFECTED SIDE TO EMPTY THUS NO ENGORGEMENT
E. USE TOPICAL LANOLIN TO MOISTURIZE

5. Which of the following would be an indication to start single agent chemotherapy in H mole cases?
A. A rise in hCG titer
B. A plateau of hCG titers for 1 week
C. Appearance of liver metastasis
D. Return of hCG titer to normal 6 weeks after evacuation

6. What is the discriminatory BHcg level above which failure to visualize an intrauterine pregnancy likely
indicates that a pregnancy is not alive or is ectopically located?
A. >1500 mIu/ml
B. >1000 mIU/ml
C. >500 mIu/ml
D. >100 mIu/ml
7. Footdrop due to prolonged lithotomy position on a stirrup is an injury to what nerve?
A. iliohypogastric
B. ilioinguinal
C. lumbosacral
D. lateral femoral
E. Peroneal

8. Placenta accreta ultrasound and colour doppler shows the following pathognomonic features:
A. a distance less than 1 mm between the uterine serosa-bladder interface and the retroplacental
vessels
B. positive color doppler result :abnormal areas of hypervascularity with dilated blood vessels within
the placental and uterine tissues
C. identification of large intraplacental lakes
D. all of the choices

9. WHAT BEST DEFINE POST PARTUM HEMORRHAGE?


A. Blood loss: 500 ml or more within 24 hours after vaginal birth
B. Blood loss of 1000 ml or more within 24 hours after cesarean birth
C. All of the choices
D. Any time the postpartum hematocrit decreases one from baseline

10. TRUE ABOUT REPRODUCTIVE TRACT INVOLUTION


A. vaginal outlet gradually regains the nulliparous dimensions
B. birth canal return to nonpregnant state after 6 weeks
C. hymen becomes small tags of tissue which scar to form the myrtiform caruncles
D. rugae reappear as prominent as before

11. A 34 year old G4P3 (3003) at 31 weeks gestation with a known placenta previa presents to the hospital
with vaginal bleeding. On assessment, she has normal vital signs, a reactive fetal heart tracing and no
uterine contractions. Heavy vaginal bleeding is noted. Which of the following is a risk factor for placenta
previa?
A. History of D&C
B. Uterine fibroids
C. Multiparity
D. Nulliparity

12. All except which of the following are associated with primary uterine rupture?
A. Prior cesarean delivery
B. Hydramnios
C. Forceps delivery
D. Breech extraction

13. Inappropriately rising serum B hCG levels only indicate a dying pregnancy, not its location. With a robust
uterine pregnancy, serum B hCG levels should increase by which of the following percentage ranges every
48 hours?
A. 63-76%
B. 53-66%
C. 73-86%
D. 23-46%

14. What is the most important criterion for the diagnosis of puerperal pelvic infection?
A. foul smelling vaginal discharge
B. fever
C. vaginal bleeding
D. Chills

15. A 30 year old G1P0 at 25 weeks presents to your clinic complaining of irregular uterine contractions
and back pain. She reports no leakage of fluid from her vagina, but says that earlier in the day she had some
very light vaginal bleeding which has now resolved. She has no prenatal care. She is dated by a sure LMP.
On arrival to the labor room, she is placed on a fetal monitor, which demonstrates uterine contractions
every 2-4 minutes. She is afebrile and her vital signs are normal. Her gravid uterus is non tender and
measures 25 cm, consistent with her gestational age by LMP. Which of the following is the most
appropriate first step in the evaluation of her vaginal bleeding?
A. Vaginal examination to determine cervical dilatation
B. Urine culture to evaluate urinary tract infection
C. Laboratory tests to evaluate for disseminated intravascular coagulation
D. Ultrasound to assess the placental location

16. Although moderate caffeine consumption is unlikely to cause spontaneous abortion, studies
consistently warn that use above what daily amount of caffeine increases the risk for miscarriage?
A. 200 mg (2 cups of coffee)
B. 400 mg (4 cups of coffee)
C. 500 mg (5 cups of coffee)
D. 300 mg (3 cups of coffee)

17. A 21 year old woman presents to the emergency room with acute onset of left lower abdominal pain
that started several hours prior to admission. She describes the pain as crampy, and intermittent. Her last
normal menstrual period was 6 weeks ago, and she reported spotting several days prior. Physical
examination reveals a tender lower abdomen with guarding, no rebound nor vaginal discharge or cervical
motion tenderness, a slightly enlarged uterus, and no palpable adnexal masses. Her blood pressure is
110/70 mmhg and pulse rate of 80 beats/minute without orthostatic changes. Which of the following is
the best appropriate diagnostic test?
A. Complete Blood Count
B. Progesterone level
C. Abdominal pelvic CT scan
D. Human chorionic gonadotropin ( HCG)

18. WHAT IS THE FIRST SURGICAL TREATMENT IN UTERINE ATONY?


1 point
A. SUTURE COMPRESSION
B. BILATERAL UTERINE ARTERY LIGATION
C. BILATERAL HYPOGASTRIC ARTERY LIGATION
D. HYSTERECTOMY

19. SIGNS AND SYMPTOMS OF BLADDER DYSFUNCTION AFTER DELIVERY


A. bladder overdistention
B. inability to spontaneously urinate within 4 hours
C. urinary retention with more than 200 mL
D. all of the choices

20. WHAT IS LIGATED IN BILATERAL HYPOGASTRIC ARTERY LIGATION?


A. EXTERNAL ILIAC ARTERY
B. COMMON ILIAC ARTERY
C. UTERINE ARTERY
D. INTERNAL ILIAC ARTERY
E. OVARIAN ARTERY

21. Which of the following gestational ages and weights are typically used to define abortions?
A. Less than 16 weeks, less than 500 grams
B. Less than 12 weeks, less than 100 grams
C. Less than 20 weeks, less than 500 grams
D. Less than 12 weeks, less than 250 grams

22. WHAT IS THE MOST COMMON CAUSE OF OBSTETRICAL HEMORRHAGE?


A. LACERATION AND HEMATOMA
B. RETAINED PLACENTAL TISSUES
C. UTERINE ATONY
D. PLACENTA ACCRETA

23. TO AVOID URETERAL INJURY IN BILATERAL UTERINE ARTERY LIGATION, WHERE SHOULD THE
LIGATURE BE APPLIED?
A. 2-3 cm above the level of uterine incision & medial to the uterine vessel
B. 2-3 cm below the level of uterine incision & lateral to the uterine vessel
C. 2-3 cm below the level of uterine incision & medial to the uterine vessel
D. 2-3 cm above the level of uterine incision & lateral to the uterine vessel

24. JJ JUST SUCCESSFULLY DELIVERED VAGINALLY, AT WHAT LEVEL SHOULD YOU EXPECT A
POSTPARTUM CONTRACTED UTERUS ?
A. ABOVE SYMPHYSIS PUBIS
B. MID UMBILICUS AND SYMPHYSIS PUBIS
C. LEVEL OF THE UMBILICUS
D. ABOVE THE UMBILICUS

25. What is the purpose of performing a dilatation and curettage prior to administering methotrexate?
A. To avoid the heavy vaginal bleeding provoked by methotrexate
B. To confirm the absence of trophoblastic tissue
C. To confirm a secretory endometrium
D. To assess for endometrial decidualization

26. A healthy 32 year old G2P1 (1001) presents to the ER at 30 weeks gestation reporting a small amount
of bright red color per vagina which occurred shortly after intercourse. It started off as spotting and then
progressed to a light bleeding. She reports no contractions, but admits to occasional abdominal cramping.
She was dated by an 18 week ultrasound, and her pregnancy has been uncomplicated. Her obstetric history
is significant for a previous low transverse cesarean at term. Vital signs are normal. Tocodynamometer
shows contractions every 10-15 minutes, and the fetal heart rate tracing is reactive. Which of the following
diagnosis may be excluded as the most likely cause for her vaginal bleeding?
A. Placenta Previa
B. Preterm Labor
C. Placental abruptio
D. Vasa Previa

27. AFTER PAINS ARE ABNORMAL IN/ WHEN:


A. BREASTFEEDING
B. MULTIPARAS
C. PRIMIPARAS
D. UTERINE INFECTIONS
28. A patient comes to your office with her last menstrual period 4 weeks ago. She denies any symptoms
such as nausea, fatigue, urinary frequency or beast tenderness. She thinks that she may be pregnant
because she has not had her period yet. She is very anxious to find out because she has a history of a
previous ectopic pregnancy and wants to be sure to get early prenatal care. Which of the following actions
is most appropriate at this time?
A. Listen for fetal heart tones by doppler
B. Perform an abdominal ultrasound
C. No action is needed because the patient is asymptomatic, has not missed her period, and cannot
be pregnant
D. Order a serum quantitative pregnancy test

29. TRUE ABOUT BILATERAL UTERINE ARTERY LIGATION


A. uterus may have necrosis due to no collateral circulation
B. recanalization of the uterine vessels will not be established within 6-8 weeks
C. reduces the pulse pressure to the uterus by 90%
D. low success rate

30. For a woman measuring 5’0” and 120 lb, what is her expected pregravid blood volume?
A. 3250 ml
B. 3800 ml
C. 3000 ml
D. 3500 ml

31. How is a low lying placenta defined?


A. The placental edge covers the internal os, but not completely
B. The placenta is implanted in the lower uterine segment - trans
C. The placental does not cover the internal os but lies within a 1 cm wide perimeter
D. The placental edge does not cover the internal os but lies within the 2 cm wide perimeter

32. A 32 year old G5P1 presents for her 1st prenatal visit. A complete obstetrical, gynecological and medical
history and physical examination is performed. Which of the following would be an indication for elective
cerclage placement?
A. Twin pregnancy
B. History of loop electrosurgical excision procedure for cervical dysplasia
C. Three spontaneous first trimester abortion
D. Three second trimester pregnancy losses without evidence of labor or abruption - quizlet

33. Maternal factors that contribute to euploid abortion include all except which of the following?
A. Frequent alcohol use
B. Dietary nutrient deficiency
C. Poorly controlled diabetes mellitus
D. Daily tobacco smoking - trans

34. The patient above undergoes suction curettage for management of H mole. The pathology report
reveals trophoblastic proliferation and hydropic degeneration with the absence of vasculature; no fetal
tissue is identified. A chest x ray is negative for any evidence of metastatic disease. Which of the following
is the best next step in her management?
A. Hysterectomy
B. Combination chemotherapy
C. Single Agent chemotherapy
D. Weekly human chorionic gonadotropin (hCG) titers - pretest obs&gyne q119 p82

35. A BREAST CONDITION WITH OBSTRUCTED BREAST DUCT BY INSPISSATED SECRETIONS.


A. Galactocele - williams page 675
B. polymastia
C. agalactia
D. polythelia
E. inverted nipples

36. What is the most appropriate treatment for a hemodynamically stable patient diagnosed with an
unruptured ectopic pregnancy via UTZ without evidence of fetal heart activity and with a hCG of 2000
mIU/ml?
A. Laparoscopic salpingostomy
B. Laparoscopic salpingectomy
C. Intramuscular MTX - trans
D. Oral Methotrexate (MTX)

Question 37 to 44:
MATCH THE PROCEDURES AND TIMING OF THE PROCEDURE
A. AFTER 6 HOURS FROM BIRTH-
B. FIRST 30 SECONDS OF LIFE-
C. 1 TO 3 MINUTES AFTER BIRTH-
D. 1 MINUTE AFTER DELIVERY-
E. 20-90 MINUTES AFTER BIRTH-
F. 5-30 MINUTES
G. 6-12 HOURS-
H. 15-20 MINUTES AFTER BIRTH
37. UTERINE PACKS- 15-20 MINUTES AFTER BIRTH
38. BIMANUAL UTERINE COMPRESSION- 5-30 MINUTES
39. DRYING THE BABY- FIRST 30 SECONDS OF LIFE
40. POST PARTUM UTERINE OXYTOCIN- AFTER 6 HOURS FROM BIRTH
41. BATHING THE MATURE BABY- 6-12 HOURS
42. DELAYED CORD CLAMPING- 1 TO 3 MINUTES AFTER BIRTH
43. SKIN TO SKIN CONTACT TO BREAST FEEDING-1 MINUTE AFTER DELIVERY
44. BREASTFEEDING- 20-90 MINUTES AFTER BIRTH

45. A 30 year old G2P1 presents at 6 weeks gestation by last menstrual period complaining of pelvic pain
and nausea. Her BHcg is 3010 mIu/ml, and no intrauterine pregnancy is seen on ultrasound. No adnexal
masses or free fluid are visualized. What is the best management strategy?

A. Surgical therapy
B. Methotrexate injection
C. No intervention
D. Expectant management with 48 hour follow up

46. GT, JUST GAVE BIRTH HOWEVER SHE SUDDENLY COMPLAINED OF ACUTE HYPOGASTRIC PAIN AND
NOTED DIAPHORESIS AND TACHYCARDIA. WHAT IS THE PRIMARY CONDITION TO CONSIDER?

A. UTERINE INVERSION
B. CERVICAL LACERATION
C. VULVAR HEMATOMA
D. UTERINE RUPTURE
Question 47 to 49:
STANDARD TIMING AFTER DELIVERY
47. PLACENTAL DELIVERY- 5-30 MINUTES
48. PUERPERIUM- 6 WEEKS
49. 4TH STAGE OF LABOR- 1-2 HOURS
A. 6 WEEKS
B. 5-30 MINUTES
C. 1-2 HOURS

Question 50 to 53:
USUAL OCCURENCE OF POST PARTUM COMPLICATIONS
50. within 4 hours
51. 3 to 5 days
52. 2 to 3 days
53. 5th day and beyond
A. post partum breast engorgement peaks- 2 to 3 days
B. postpartum blues self-limited to-
C. post partum wound dehiscence-
D. post partum urinary retention has not voided-

54.WHICH OF THE FOLLOWING BLOOD COMPONENTS/PARAMETERS IS/ARE INCREASED DURING THE


PUERPERIUM?
A. PLATELET
B. LYMPHOCYTES
C. HEMATOCRIT
D. EOSINOPHILS
E. ALL OF THE CHOICES

55. Which of the following chromosomal anomalies is most frequently identified with 1st trimester
abortion?
A. Balanced Robertsonian translocation
B. Triploidy
C. Autosomal Trisomy
D. Monosomy X

56. Using transvaginal sonography, a gestational sac is usually first visible between which of the following
weeks of gestation?
A. 5 and 6 weeks
B. 5.5 and 6 weeks
C. After 6 weeks
D. 4.5 and 5 weeks

57. TRUE ABOUT LACTATION AND BREASTFEEDING


A. IN GENERAL BREASTFEEDING IS CONTRAINDICATED IN HIV INFECTED MOTHERS
B. LACTATION DEPENDS ON THE BREAST SIZE
C. COLOSTRUM IS IMPORTANT AND INFANT BENEFITS AS LONG AS IT FEEDS
D. LATCH THE BABY'S MOUTH ON THE NIPPLE ONLY

58. Bilateral ligation of ovarian arteries is done at what site?


A. 2-3 cm above the level of uterine incision & lateral to the uterine vessel
B. at the entry of ovarian arteries at the infundibulopelvic ligaments
C. at the anastomosis between the uterine and ovarian arteries near the cornua of the uterus
D. 2-3 cm below the level of uterine incision & medial to the uterine vessel

59. HOW IS PLACENTA ACCRETA IS THE DEFINITIVELY DIAGNOSED?


A. > 2.5 MOM MSAFP
B. CT SCAN AND OR MRI
C. COLOR DOPPLER ULTRASOUND
D. HISTOPATHOLOGY

60.PREGNANCY INDUCED HYPERVOLEMIA NORMALIZES BY WHICH OF THE FOLLOWING PHYSIOLOGY?


A. DECREASED ARTERIAL STIFFNESS
B. ALL OF THE CHOICES
C. BLOOD LOSS DURING DELIVERY
D. LOW SYSTEMIC VASCULAR RESISTANCE
E. POST PARTUM DIURESIS

61. Which of the following carries the highest risk of subsequent ectopic pregnancy?
A. Positive test result for cervical Chlamydia trachomatis
B. Prior ectopic pregnancy
C. Smoking
D. More than five lifelong sexual partners

62.TRUE ABOUT DECIDUA & ENDOMETRIAL REGENERATION


A. endometrial regeneration is rapid, except at the placental site
B. spongy & basalis layer are sloughed off
C. Option 5
D. necrotic basalis is sloughed off in the lochia
E. histological endometritis reflects infection

63. The above patient continues to bleed heavily and you observe persistent late decelerations on the fetal
heart tracing. Her blood pressure and pulse are normal. You explain to the patient that she needs to be
delivered, and she is delivered by cesarean section under general anesthesia. The baby and placenta are
easily delivered, but the uterus is noted to be boggy and atonic despite intravenous infusion of Oxytocin.
Which of the following is contraindicated in this patient for the treatment of uterine atony?
A. Misoprostol suppositories
B. Terbutaline administered intravenously
C. Prostaglandin F2a suppositories
D. Methylergonovine intramuscularly

64. On the time of laparoscopy for an ectopic pregnancy, there was note of a 3 cm mass in the ampulla of
the left fallopian tube, consistent with an unruptured ectopic pregnancy. There is no blood in the cul de sac.
What is the best next step in management?
A. Now that you have confirmed the diagnosis, you should leave the fallopian tube alone, and
recommend treatment with Methotrexate
B. Perform a laparoscopic salpingectomy
C. Perform a laparoscopic salpingectomy and recommend postoperative treatment with
Methotrexate
D. Perform a laparoscopic salpingostomy

65. LS, 25 years old who just delivered vaginally and was not able to expel the placenta from the uterus
within 30 minutes after the delivery of the baby .What is the diagnosis?
A. POST PARTUM HEMORRHAGE
B. UTERINE SUBINVOLUTION
C. UTERINE ATONY
D. RETAINED PLACENTA

66. Which of the following is the best next step in the management of this patient?
1 point
A. Administer intramuscular terbutaline
B. Administer methylergonovine
C. Admit and stabilize the patient
D. Perform cesarean delivery

67. JK, G1P1 JUST DELIVERED VAGINALLY BY A CLERK AND NOTED UTERINE INVERSION WITH
PLACENTA STILL ATTACHED.WHAT SHOULD BE DONE?
A. DO NOT REMOVE PLACENTA AND GIVE OXYTOCIN TO PREVENT FURTHER HEMORRHAGE
B. REMOVE PLACENTA AND GIVE OXYTOCIN TO PREVENT FURTHER HEMORRHAGE
C. DO NOT REMOVE PLACENTA AND REPOSITION UTERUS AT ONCE
D. REMOVE PLACENTA AND REPOSITION UTERUS AT ONCE

68. WHICH OF THE FOLLOWING CAUSES SECONDARY POSTPARTUM HEMORRHAGE?


A. VASA PREVIA
B. PLACENTAL POLYP
C. ABRUPTIO PLACENTA
D. PLACENTA ACCRETA

69. A 24 year old woman who is 7 weeks pregnant came to your clinic for prenatal care. She brought a
transvaginal ultrasound which failed to reveal an intrauterine gestation. The endometrial lining is 4 mm, the
ovaries are normal, and there is a 1.5 cm mass adjacent to the right ovary. There is no gestational sac, yolk
sac, or embryo seen in the uterus and the adnexae, and no fluid is seen in the cul de sac. Her hCG level is
4500 mIU/ml. What is the patient’s most likely diagnosis?
A. Ectopic Pregnancy
B. Incomplete Abortion
C. Missed Abortion
D. Complete Abortion

70. A 19 year old G1P0 presents for her first prenatal visit. She is 12 weeks pregnant by sure last menstrual
period. She reports vaginal bleeding, and on physical exam, you appreciate a 16 weeks size uterus. You are
unable to detect fetal heart tone with a doppler. Ultrasound showed Hydatidiform Mole. Which of the
following best describes the patient’s diagnosis?
A. Older maternal age is not a risk factor for hydatidiform mole
B. The most common chromosomal make up of partial mole is 46 XX, of paternal origin
C. Partial mole has a higher risk of developing into choriocarcinoma than complete mole
D. Vaginal bleeding is a common symptom of hydatidiform mole

71. Theca lutein cysts, if found bilaterally in the adnexa of a patient with molar pregnancy, increases the
risk of which of the following?
A. Gestational Trophoblastic Neoplasia
B. Thyroid storm
C. Hyperemesis gravidarum
D. Preeclampsia

72. Which of the following should always be considered in the differential diagnosis of threatened abortion?
A. Oral contraceptive use
B. Ectopic pregnancy
C. Minor maternal trauma
D. Paternal factors

73. Which of the following is typical breast engorgement treatment?


A. COMBINED ORAL CONTRACEPTIVES
B. ACETAMINOPHEN
C. CARBEGOLINE
D. BROMOCRIPTINE

74. TL, GAVE BIRTH A WEEK AGO AND ON FOLLOW UP, THERE IS STILL REDDISH LOCHIA WITH UTERUS
LARGER THAN THE EXPECTED SIZE.NO OTHER SIGNS AND SYMPTOMS.WHEN ASKED ABOUT
BREASTFEEDING, SHE PRACTICES MIXED FEEDING.AT THIS POINT, WHICH OF THE FOLLOWING
PUERPERAL COMPLICATIONS MOST LIKELY WILL BE ANTICIPATED?
A. PUERPERAL SEPSIS
B. UTERINE SUBINVOLUTION
C. UTERINE ATONY
D. OBSTETRICAL HEMORRHAGE

75. With regard to molar pregnancies, what does the term androgenesis refer to?
A. Development of a zygote that contains only maternal chromosomes
B. Development of theca lutein cysts
C. Development of a zygote that contains only paternal chromosomes
D. Absence of fetal tissue and amnion

76. WHICH OF THE FOLLOWING IS/ARE PREVENTIVE OF PUERPERAL COMPLICATIONS?


A. PERINEAL CARE FROM POSTERIOR TO ANTERIOR
B. EARLY AMBULATION PREVENTS DVT
C. DIETARY RESTRICTIONS TO AVOID CONSTIPATION
D. ALL OF THE CHOICES
E. PER DEMAND BREASTFEEDING TO PREVENT NIPPLE FISSURES

77. IDENTIFY PROCEDURE TO TREAT UTERINE ATONY

Bimanual Compression

78. In the treatment of ectopic pregnancy, laparoscopy has what advantage over laparotomy?
A. Decreased hospital stays
B. Lower fertility rate
C. Lower ectopic pregnancy rate
D. Comparable persistent ectopic tissue rate

79. A 39 year old G5P3 presents at 6 weeks gestation with lower abdominal pain. A transvaginal ultrasound
is performed for further evaluation. What findings would be expected on transvaginal ultrasound if her
dates are correct and her pregnancy is viable?
A. Gestational sac only
B. Gestational sac and yolk sac
C. Gestational sac, yolk sac, and fetal pole with cardiac motion
D. Gestational sac, yolk sac, and fetal pole without cardiac motion

80. A 32 year old G2P1 at 28 weeks gestation presents to the emergency room with a chief complaint of
vaginal bleeding. Her vital signs are: BP: 120/70 mmhg, pulse of 87 beats/minute and temperature 37.0C.
She reports no contraction and states that the baby is moving normally. On ultrasound, the placenta is
located on the anterior wall of the uterus, and completely covers the internal cervical os. Which of the
following would most increase her risk for hysterectomy?
A. Desire for sterilization
B. Development of Disseminated Intravascular Coagulopathy (DIC)
C. Placenta accreta
D. Prior vaginal delivery

81. KM, G4P3(0300) IS NEAR TERM AND HAD PREVIOUS 3 CS OPERATIONS.ULTRASOUND REVEALED
ANTERIORLY IMPLANTED LOW LYING PLACENTA AND WITH BLOOD LAKES.WHAT ANTICIPATORY
SURGICAL TREATMENT SHOULD BE ADVISED?
A. CS 4 WITH FOCAL REMOVAL OF PLACENTA, UTEROTONICS AND METHOTREXATE
B. CS 4 WITH POSSIBLE HYSTERECTOMY
C. CS 4 WITH BTL
D. CS 4 , LEAVE THE PLACENTA AND METHOTREXATE

82. What progesterone value threshold is most helpful to exclude ectopic pregnancy?
A. >25 ng/ml(book)
B. >10 ng/ml
C. >15 ng/ml
D. >20 ng/ml (If <20 probable ectopic preg and intrauterine preg)

83. Which of the following is a safe breastmilk to feed?


A. expressed milk of a mother on radio iodine therapy
B. expressed milk from a mother with inguinal active herpes simplex virus
C. expressed milk at room temperature for 8 hours
D. expressed milk from a mother with history of treated tuberculosis

84. WHAT IS THE MANEUVER TO AVOID UTERINE INVERSION?


A. MC ROBERT'S
B. MODIFIED BRANDT-ANDREWS
C. CORKSCREW
D. GASKIN'S
E. JOHNSON MANEUVER

85. A patient presents for her first OB visit after having a positive home pregnancy test. She reports her last
menstrual period 8 weeks ago, but she is not entirely certain because she has a long history of irregular
menses. Her urine pregnancy test in your clinic is positive. Which of the following is the most accurate way
to date this patient’s pregnancy?
A. Determination of uterine size on pelvic examination
B. Quantitative serum hCG
C. Determination of progesterone level along with serum hCG level
D. Crown rump length on abdominal or vaginal ultrasound

86. WHICH OF THE FOLLOWING IS AN ABSOLUTE CONTRAINDICATION FOR BREASTFEEDING?


A. on breast cancer treatment
B. active herpes simplex virus
C. hepatitis C infection
D. hepatitis B virus infection

87. Couvelaire uterus is seen in:


A. Myoma in Pregnancy
B. Abruptio Placenta
C. Placenta Previa,
D. Pelvic Inflammatory Disease

88. Most ectopic pregnancies implant in the fallopian tube. Where is the least common fallopian tube
implantation site?
A. Fimbria
B. Ampulla
C. Isthmus
D. None of the above

89. What is the cellular mechanism of action of methotrexate?


A. DNA intercalation
B. Impedance of DNA and RNA synthesis
C. Inhibition of microtubule formation
D. Alkylation of proteins, DNA and RNA

90. KL, GAVE BIRTH 1 WEEK AGO AND WHAT KIND OF LOCHIA SHOULD BE OBSERVED ON HER?
A. LOCHIA SEROSA
B. ALL OF THE CHOICES
C. LOCHIA RUBRA
D. LOCHIA ALBA

91. At what lower limit serum B hCG concentration is selected to represent the discriminatory value of
which transvaginal sonography can reliably visualize pregnancy?
A. 5500 or 6000 IU/L
B. 2500 or 3000 IU/L
C. 4500 or 5000 IU/L
D. 1500 or 2500 IU/L

92. Causes of uterine atony include which of the following?


A. Placenta previa
B. Obesity
C. Placental abruption
D. Multiple fetuses

93. IDENTIFY WHICH ONE IS PAROUS CERVIX


B

94. A 19 year old G1 is diagnosed with a 2.3 cm right ectopic pregnancy. Her BHcg is 1967 mIU/ml, her
hematocrit is 37%, and she has a small amount of free fluid in her cul de sac. She strongly desires expectant
management rather than immediate treatment. Which aspect of her history favors successful resolution
with expectant management?
A. BHcg <2000 mIU/ml
B. Ectopic pregnancy size <3 cm
C. Free fluid in the cul de sac
D. Hematocrit above 35%

95. WHAT IS THE COMPRESSION SUTURE OF CHOICE IN UTERINE ATONY AFTER VAGINAL DELIVERY?
A. B-LYNCH
B. CHO
C. HAYMAN
D. MARASINGHE
E. MATSUBARA-YANO

96. What is the most common etiologic factor for ectopic pregnancy?
A. History of Pelvic Inflammatory Disease (PID)
B. Prior abortion
C. Prior tubal surgery
D. Genetically abnormal embryos

97. A patient at 17 weeks gestation is diagnosed with an intrauterine fetal demise. She desires expectant
management. She returns to your clinic 5 weeks after and her vital signs are: BP: 110/70 mmhg, pulse of
93 beats/minute, 36.3C and respiratory rate of 16 breaths/minute. She has not has a miscarriage, although
she has had some occasional spotting. Her cervix is closed on examination. This patient is at increased
risk for which of the following?
A. Septic abortion
B. Future infertility
C. Recurrent abortion
D. Consumptive coagulopathy

98. TRUE ABOUT MEDICAL TREATMENT OF POST PARTUM HEMORRHAGE :


A. PRBC: platelet concentrate in a ratio of 10: 6 units
B. IV fluids: blood loss in a ratio of 3:1 L
C. PRBC:FFP in a ratio of 2:1 units
D. All of the choices

99. Your patient is a 22 year old G2P1 at 36 weeks gestation with a history of prior abruption during a
spontaneous vaginal delivery. Which of the following might be effective for preventing recurrence during
this pregnancy?
A. Antepartum testing
B. Delivery at 38 weeks
C. Cesarean delivery at 39 weeks
D. Cesarean delivery at 38 weeks

100.. TRUE ABOUT IMMEDIATE POST PARTUM UTERUS


A. lower segment is converted to uterine isthmus
B. all of the choices
C. uterus weighs approximately 100g
D. anterior and posterior walls, which lie in close apposition, are each 4 to 5 cm thick

101. A 32 year old woman presents to the emergency room with abdominal pain and vaginal bleeding. Her
LMP was 8 weeks ago, and her pregnancy test is positive. Her blood pressure is 80/60 mmhg, pulse rate
of 110 beats/minute. On examination, her abdomen is distended and tender. An abdominal UTZ showed
fluid within the abdominal cavity. The decision is made to take the patient to the operating room for
emergency exploratory laparotomy. Which of the following is the most likely diagnosis?
A. Torsed ovarian corpus luteal cyst
B. Ruptured ectopic pregnancy
C. Hydatidiform mole
D. Incomplete abortion
LONG EXAMINATION -2
(2021 - 2022)
SURE
UNSURE

DR. DE GUZMAN

1. A G1P0 38 wks AOG, was admitted with regular uterine contractions, with cervix dilated
to 4 cms on the 10th hour of labor (7 am). Repeat IE at 9 am revealed cervical dilatation
of 5 cms, and at 11 am cervix was dilated to 6 cms, cephalic station -3. What labor
abnormality is evident at this point?
A. prolonged latent phase
B. protracted active phase of dilatation
C. arrest of cervical dilatation
D. labor is progressing normally

2. A pathological retraction ring of Bandl:


A. associated with marked thinning and stretching of the LUS
B. signifies impending uterine rupture
C. seen as a uterine indentation
D. all of the above

3. The sidewalls of an android pelvis:


A. parallel and straight
B. parallel
C. convergent
D. Straight

4. What is a possible adverse effect on the fetus with failure to retract the prolapsed hand
in a compound presentation? *
A. duchenne brachial palsy
B. cerebral palsy
C. prolonged descent
D. ischemic necrosis of the presenting arm

5. A diagnosis of CPD may be made with this obstetric finding: *


A. unengaged head of a primigravida at term
B. deflexed attitude at the onset of labor
C. both
D. Neither
6. Parturient in #5 above was reassessed at 12 pm and cervical dilatation was 8 cms,
cephalic, station +1. After 3 hrs, IE findings were the same. What is the labor abnormality?
*
A. protracted active phase
B. arrest of dilatation and descent
C. failure of descent
D. labor is progressing normally

7. The preferred management of the above case is: *


A. send the patient home to wait for more frequent contractions
B. admit the patient and perform cesarian section
C. advise patient to ambulate
D. advise bed rest

8. For a neglected transverse lie, internal podalic version and extraction can be
considered if: *
A. intrapartal death of a term baby
B. cesarian is not feasible
C. infection is severe and widespread
D. membranes are intact

9. Reevaluation with clinical pelvimetry of the parturient in #5 above showed a bituberous


dm <8cm. What is the preferred management ? *
A. reassess after 2 hours and await spontaneous vaginal delivery
B. do fundal pressure
C. deliver by cesarian section
D. deliver by midforceps extraction

10. The caput succedaneum that forms from extreme molding of the fetus in the above
case can be found at the: *
A. sinciput
B. parietal area
C. temporal area
D. forehead

11. A 16 year old primigravid was seen at the ER. She is 138 cm tall, walks with a waddle,
and was noted to have pectus carinatum. You suspect that she may have a contracted
pelvis. What would you advise her regarding labor and delivery? *
A. she will deliver by outright cesarian section
B. she will go into trial of labor
C. all short women have a contracted pelvis
D. her baby cannot be delivered vaginally

12. What is an extremely heroic type of treatment for the case? *


A. Magnesium Sulfate
B. Isoflurane
C. oxytocin
D. isoxsuprine
13. Placing the fist between the ischial tuberosities approximately measures the: *
A. pelvic inlet
B. midpelvis
C. pelvic outlet
D. all of the above

14. This pelvic diameter is measured from the sacral promontory to the nearest point on
the posterior surface of the symphysis pubis: *
A. true conjugate
B. obstetric conjugate
C. diagonal conjugate
D. it is equivalent to 12 cms.

15. Foul-smelling meconium draining from the mother’s vagina, concentrated urine,
edema of the vulva and cervix in a woman who has been pushing for a long time, are
signs that suggest: *
A. chorioamnionitis
B. recto vaginal fistula
C. uterine atony
D. cephalopelvic disproportion

16. Regarding the case in #41, you explain to the patient that symptom will resolve within:
* 41. After giving birth to her first-born, the patient complained of numbness of the lower extremity. This must
be due to:
A. 6 weeks
B. 2 months
C. 3 months
D. 6 months

17. The incidence of difficult deliveries is increased when: *


A. AP diameter of the pelvic inlet is 10 cm
B. Transverse diameter is < 12 cm
C. both
D. neither

18. Midpelvic dimensions can be inferred if: *


A. spines are not prominent
B. pelvic sidewalls are parallel ~converge
C. sacrosciatic notch is wide ~narrow
D. intertuberous diameter is >10cm ~<8cm

19. The answer above has this presenting diameter: * (jumbled questions)
A. verticomental equivalent to 13.5 cm (prev marked answer)
B. biparietal equivalent to 10 cm
C. submentobregmatic equivalent to 9.5 cm
D. none of the above
20. If the pubic arch of the pelvic outlet is very wide, this must be a: *
A. gynecoid pelvis
B. anthropoid pelvis
C. android pelvis
D. platypelloid pelvis

21. A G3P2 on her 39th wk of gestation has been in active labor for 8hrs before delivering
a 4 Kg baby boy by a difficult vaginal delivery. Maternal complications of dystocia are: *
A. chorioamnionitis
B. uterine atony
C. abnormal thinning of the lower uterine service
D. all of the above

22. Palpation of the upper and lower poles of the uterus shows neither the head nor the
breech *
A. cephalic presentation
B. breech presentation
C. shoulder presentation
D. face presentation

23. This pelvis has a short AP diameter, long transverse diameter and a short anterior
sagittal diameter: *
A. gynecoid
B. android
C. anthropoid
D. platypelloid

24. On clinical pelvimetry of a primigravid woman in early labor, it was noted that the AP
diameter was adequate with the sacral promontory not reached at 11 cms. The midpelvis
was assessed to have reduced capacity in all diameters, with a prominent ischial spines;
while the pelvic outlet showed a narrow bituberous diameter. This parturient can be
predicted to have one of the following outcomes during labor, except:
A. deep transverse arrest as occiput posterior with failure of rotation
B. delay at the pelvic inlet
C. delivery may be accomplished by difficult forceps application, rotation and traction
D. may lead to major perineal tears on expulsion of the fetus

25. Hypotonic uterine dysfunction is characterized by: *


A. incoordinate uterine dysfunction
B. distorted pressure gradient
C. complete asynchrony of impulses from each cornu
D. no basal hypertonus

26. A full term primigravid patient started having regular contractions 4 hrs prior to
admission. Internal exam revealed cervical dilatation of 4 cms. After 2 hours, with
adequate uterine contractions, a repeat internal exam was done which revealed cervical
dilatation of 5 cms, and after another 3 hrs, cervix was noted to be 7 cms dilated. This
patient’s labor pattern is: *
A. protracted latent phase
B. protracted active phase
C. protracted descent
D. prolonged deceleration phase

27. Diffuse swelling of the scalp of the newborn with subcutaneous fluid collection and
poorly defined margins *
A. cephalhematoma
B. caput succedaneum
C. caput medusae
D. molding

28. A 25 yr old G1P0 37-38 wks AOG, was brought to the ER because of on and off
hypogastric pain and contractions which started around 2 hrs prior to consult. On IE, the
cervix was about 1 cm dilated, 30% effaced. She was advised go home and come back
when contractions become more frequent. Patient returns to the ER after a day, now
complaining of more frequent hypogastric contractions. On IE, cervix was 2 cms dilated,
50% effaced, cephalic presentation, station -3 and no bloody show. What is the labor
abnormality in this case? *
A. none; she is in the latent phase of labor
B. protracted labor
C. prolonged latent phase
D. arrest of cervical dilatation

29. If the fetal chin is felt anteriorly towards the symphysis, and no cephalopelvic
disproportion is evident, what is the management? *
A. cesarian section
B. outlet forceps delivery
C. vacuum extraction
D. trial of labor for normal vaginal delivery

30. Prerequisites for the diagnosis and management of a neglected transverse lie, except:
*
A. prolonged labor with ruptured membranes
B. fetal impaction with a prolapsed cord
C. vaginal delivery should be attempted with a dead baby
D. lower uterine segment is stretched and thin

31. what is the position of the fetal head shown in the picture?
A. left occiput posterior
B. right occiput posterior
C. right occiput anterior
D. left occiput anterior

32.The presenting diameter of the fetal presenting part in the case in #11 is: *
A. submentobregmatic?
B. verticomental
C. biparietal
D. scapula

33. G1P0 39-40 wks AOG in labor; FH = 38 cms, Ht = 148 cms; Clinical pelvimetry: Ischial
spines prominent; sacral promontory easily reached at 11 cms, sacrosciatic notch
narrow, pelvic sidewalls convergent. These findings indicate: *
A. contracted pelvis
B. excessive fetal size
C. both
D. neither

34. The fetal mentum was felt as the leading pole on IE of a woman in labor. What is the
presentation? *
A. transverse
B. face
C. brow
D. breech

35. The incidence of anthropoid pelvis in the general female population: *


A. 50%
B. 20%
C. 25%
D. 5%

36. There is a 4-minute shorter interval to delivery with the parturient in squatting position,
for the following reason/s: *
A. Pelvic inlet diameters increase by 20-30%
B. less intense aortocaval compression
C. Fetal presenting part moves toward the sacrum
D. Relaxation of the abdominal muscles during squatting contributes to more efficient labor
37. The following may be attempted in managing the above case: *
A. manual conversion to a vertex presentation
B. forceps or manual rotation of the chin
C. internal podalic version and extraction
D. none of the above
E. any of the above

38. A 38 yr old G6P5(5005) 39 wks AOG, was rushed to the ER due to frequent uterine
contractions which started an hour prior to consult. On IE the cervix was 5 cms dilated,
80% effaced, cephalic, station +1. After 1 hr, cervix was 8 cms dilated and gave birth by
spontaneous vaginal delivery 30 minutes later. What possible fetal complications can be
expected with this labor abnormality? *
A. intracranial trauma
B. nuchal cord loop
C. cerebral palsy
D. neonatal sepsis

39. True regarding treatment of uterine dysfunction: *


A. Direct IV administration of oxytocin as treatment for certain types of uterine dysfunction
B. utilization of midforceps delivery when oxytocin fails
C. undue labor prolongation may contribute to increased maternal and perinatal morbidity and
mortality
D. all of the above

40. What is the fetal head position shown? *

A. right occiput anterior


B. left occiput anterior
C. left occiput transverse
D. right occiput transverse

41. The cardinal movements of engagement and internal rotation occur here: *
A. pelvic inlet
B. midpelvis
C. pelvic outlet
D. pelvic rim
42. A fetus presenting with an attitude of partial extension: *
A. vertex presentation (Flexion)
B. face presentation (Hyperextension)
C. brow presentation
D. breech presentation (Feet)

43. True of treatments of first stage disorders *


A. Prolonged latent phase: if false labor, treat with observation and sedation if needed
B. Protraction disorder of active phase: augment with amniotomy and oxytocin
C. Arrest disorder with adequate contractions: CS
D. all of the above

44. The mechanism of labor for the above case : *


A. engagement, descent, flexion, internal rotation, extension, external rotation, expulsion
B. extension, descent, internal rotation, flexion, restitution, external rotation (face)
C. engagement, flexion, descent, internal rotation, external rotation, expulsion
D. none of the above

45. How should the case in #5 above be managed? *


A. bed rest
B. cesarian section CPD?
C. augment labor with oxytocin
D. start epidural anesthesia

46. What is the presenting part with a fetus shown in the illustration? *

A. face
B. brow
C. vertex
D. breech

47. A G2P1 (1001), 40 wks AOG, was in labor for 8 hrs, with cervix dilated to 7 cms, 70%
effaced, cephalic, Station 0. After 2 hrs, cervix progressed to 9 cms, still at station 0.
Oxytocin was started and after 30 minutes more, cervix became fully dilated, but
presenting part remained at station 0. Parturient was asked to push with each contraction
and after an hour, the presenting part remained at station 0. What is your diagnosis? *
A. prolonged descent
B. protracted descent
C. failure of descent
D. arrest of descent

48. The preferred management for the patient described above: *


A. CS
B. forceps delivery
C. expectant & support
D. fundal pressure

49. Which of the following statements regarding the second stage of labor is true? *
A. fetal descent largely follows complete cervical dilatation
B. It is limited to 1 hr among nulliparas, and extended to 2 hrs with regional analgesia.
C. After 2 hrs in the second stage, delivery by cesarian section increases progressively.
D. There is about 20-25% chance of spontaneous delivery when the second stage of labor
reaches 5 hrs.
E. Option 2

50. After giving birth to her first-born, the patient complained of numbness of the lower
extremity. This must be due to: *
A. compression of the common fibular nerve
B. compression of the common peroneal nerve
C. either
D. Neither

DR. NACIS
51. In case of placenta previa totalis anterior,in a previous CS II; what is your best
hysterotomy option?
A. Classical CS with possible hysterectromy
B. J incision with possible hysterectromy
C. high transverse CS with possible hysterectromy
D. Low Transverse Cesarean Section with possible hysterectromy

52. THE RECOMMENDED CS INCISION:


A. MIDLINE VERTICAL INCISION
B. PFANNENSTIEL INCISION
C. JOEL-COHEN INCISION
D. PARAMEDIAN INCISION

53. WHICH OF THE FOLLOWING IS TRUE ABOUT EPISIOTOMY?


A. RIGHT MEDIOLATERAL EPISIOTOMY TGRANSECTS THE PERINEAL BODY
B. MEDIAN EPISIOTOMY HAS THE ADVANTAGE OF PREVENTING RECTOVAGINAL FISTULA
OVE5R RIGHT MEDIOLATERAL EPISIOTOMY
C. SELECTIVE EPISIOTOMY IS RECOMMENDED
D. IT IS ROUTINE TO ALL PRIMIPARA TO AVOID EXTENSION OF THE WOUND

54. A 21 year old G2P1 (1001) came in for prenatal checkup. She is unsure of her last
menses. Her previous pregnancy was delivered by LTCS for placenta previa. The plan is
now for repeat CS for lack of consent for VBAC. When will you do the repeat CS?
A. If ultrasound measurement < 20 weeks supports a current gestation of 39 weeks or more.
B. All are correct
C. If it has been 34 weeks since a positive urinary or serum B-hCG pregnancy test was taken.
D. If fetal heart sounds have been documented for 20 weeks by Dopplers.

55. IDENTIFY THE TYPE OF BREECH

A. FRANK
B. DOUBLE FOOTLING
C. FOOTLING
D. COMPLETE

56. What is the type of forceps is recommended to be used in cases of asynclitism?


A. Simpson’s
B. Kielland’s
C. laufe
D. tucker-mcklean
E. piper

Question 57 to 59:
BREECH DELIVERY MANEUVERS
57. Delivery of the after coming head• index & middle finger applied over the maxillae to flex the head
58. 2 fingers grasping shoulders of the back- down fetus
59. Two fingers are carried up along one extremity to the knee to push it away from the midline.
Spontaneous flexion follows
A. PINARD'S
B. MAURICEAU
C. PRAGUE
PINARD'S - Two fingers are carried up along one extremity to the knee to push it away from the
midline. Spontaneous flexion follows

MAURICEAU - Delivery of the after coming head• index & middle finger applied over the maxillae to
flex the head

PRAGUE 2 - fingers grasping shoulders of the back- down fetus

60. Most common sign of uterine rupture:


A. hypotension
B. vaginal bleeding
C. non-reassuring fetal heart rate pattern(variable decelerations)
D. palpable fetal parts
E. non-reassuring fetal heart rate pattern(tachycardia)

61. In a case with “back down “ baby, what is your best hysterotomy option?
A. Low Transverse Cesarean Section (For Transverse Lie)
B. J incision
C. Classical CS
D. high transverse CS

62. On post-partum follow up, your patient who had prolonged 2nd stage of labor
complained of difficulty of walking with tingling sensation and numbness in the medial
right thigh. On examination, the patient had difficulty in thigh adduction on the right. What
nerve has been traumatized during delivery?
A. Superior gluteal nerve
B. Inferior gluteal nerve
C. Femoral nerve
D. Obturator nerve

63. This technique is employed to facilitate delivery of the entrapped aftercoming head
during breech delivery.
A. Duhrrsen incision (Can use Mauriceau Maneuver or Prague Manuver as well for the
aftercoming head)
B. Mueller-Hillis maneuver (used to assess fetal descent)
C. Ritgen maneuver
D. Kronig incision

64. TRUE ABOUT OPERATIVE OBSTETRICS


A. The most important function of FORCEPS AND VACUUM is traction.
B. All of the choices
C. Operative OB is best done if the presenting part is at the outlet
D. The goal is to shorten the 2nd stage of labor

65. Criteria for Outlet Forceps Extraction:


A. All of the options
B. Scalp is visible at the introitus upon separating the labia. (WITHOUT separation of the labia)
C. Fetal head is at station 0 (Fetal head is at or on the perineum so station +3)
D. Rotation does not exceed 90 DEGREES (Does not exceed 45 degrees)
E. Sagittal suture is in the A-P diameter or ROA,ROP, LOA, LOP

66. WHICH OF THE FOLLOWING IS AMENABLE FOR INTERNAL PODALIC VERSION?


A. BREECH PRESENTATION
B. SHOULDER PRESENTATION
C. DELIVERY OF SECOND TWIN
D. BROW OR FACE PRESENTATION
E. TRANSVERSE LIE

67. A 32 year old G3P2 (2002) came in at 32 weeks AOG with a chief complaint of
contractions. All her previous pregnancies were delivered vaginally. Her previous babies
weighed 2.9 and 3.0 kg. On abdominal exam, FH was 29 cm, EFW 1.4-1.6 kg, frank breech,
FHT 145 bpm at the right periumbilical area. On IE, the cervix was 8 cm dilated, fully
effaced, intact BOW, station 0. What is the best mode of delivery for this patient?
A. Outright abdominal delivery
B. Complete breech extraction
C. External cephalic version followed by normal vaginal delivery
D. Partial breech extraction

68. GENERAL EARLIEST POSSIBLE REMOVAL OF FOLEY CATHETER IS AFTER:


A. 24 HOURS
B. 48 HOURS
C. 12 HOURS
D. 6 HOURS (6 to 8 hours)

69. When is the recommended time to start ECV-external cephalic version?


A. 34 weeks AOG
B. 37-40 weeks AOG (Performed after 37 weeks AOG)
C. 28 weeks AOG (Never before 34 weeks AOG)
D. 36 weeks AOG

70. FG, A MULTIGRAVID CAME IN DUE TO ENTRAPPED FETAL HEAD AFTER FETAL BODY
WAS DELIVERED IN A TRICYCLE WHILE IN TRANSIT TO R1MC.THE ATTENDING OB
DECIDED TO DO DUHRSSEN'S CERVICAL INCISION, WHERE IS THE IDEAL POSITION TO
DO SO?
A. 12 ,3, 6 AND 9 O'CLOCK
B. 9 AND 3 O'CLOCK
C. 12 AND 6 O'CLOCK
D. 10, 2 AND 6 O'CLOCK

71. TRUE ABOUT THE OPERATIVE OBSTETRIC INSTRUMENT APPLICATION ON THE


FETAL HEAD
A. THE CEPHALIC CURVATURE OF THE FORCEPS SHOULD BE IN APPOSITION WITH THE
BIEPARIETAL AREA
B. THE BLADES GRASP ALONG THE AXIS OF THE OCCIPOTOMENTAL DIAMETER
C. THE BLADES SHOULD BE IN EQUIDISTATNT ALONG THE SAGITTAL SUTURE
D. ALL OF THE CHOICES
E. VACUUM CUP SHOULD BE APPLIED 3 CMS ANTERIOR TO OCCIPUT

72. PROLONGED 2ND STAGE OF LABOR IS DEFINED AS:


A. NULLIPARA WITH ANALGESIA: >/=3 HOURS
B. MULTIPARA WITH ANALGESIA:>/=3 HOURS

73. FG, HAD 22 HOUR GRUELING LABOR IS NOW COMPLAINING OF SEVERE PAIN UPON
SITTING.WHAT NERVE COULD HAVE BEEN SEVERED?
A. Inferior gluteal nerve
B. Pudendal nerve
C. Sciatic nerve
D. Superior gluteal nerve

74. SD, IS IN LABOR WITH BREECH PRESENTATION AND WITH RUPTURED BAG OF W
H OF THE FOLLOWING WOULD ONE SHOULD WORRY MOST OF CORD PROLAPSE
OCCURENCE?
A. COMPLETE
B. DOUBLE FOOTLING
C. FRANK
D. FOOTLING (15%)

75. WHICH OF THE FOLLOWING IS TRUE ABOUT THE MANAGEMENT OF BREECHES?


A. ALL OF THE ABOVE
B. EPIDURAL ANESTHESIA IS EFFECTIVE DURING EXTERNAL CEPHALIC VERSION
C. FETAL BLOOD SAMPLING FROM THE FETAL BUTTOCKS CAN PROVIDE ACCURATE ACID
BASE STATUS
D. X RAY PELVIMETRY IS IMPORTANT FOR THE ASSESSMENT OF SUCCESSFUL VAGINAL
DELIVERY

76. WHICH OF THE FOLLOWING DOES NOT APPEAR TO INCREASE THE RISK OF TRIAL
OF LABOR COMPLICATIONS?
A. Fetal death
B. EFW of more than 4000 grams
C. Interdelivery interval ≤ 18 months
D. Twin pregnancy
E. Maternal obesity

77. IDENTIFY
Complete Vacuum Delivery System

78. TO LESSEN POST OP ADHESIONS, WHICH OF THE FOLLOWING IS RECOMMENDED?


A. prevent infection
B. achieve hemostasis
C. all of the choices
D. minimize tissue ischemia
E. handle tissues delicately

79. WHICH OF THE FOLLOWING IS TRUE ABOUT TIMING OF INDUCTION OF LABOR?


A. ALL OF THE CHOICES
B. PD, WITH PREECLAMPSIA AND ON METHYLDOPA AT 37 WEEKS AOG
C. KT, WITH PROM 2 HOURS AGO
D. GR, WTH DM AND ON INSULIN AT 39 WEEKS AOG
E. TG, 42 WEEKS NOT IN LABOR

80. Which of the following is an indication for total breech extraction?


A. all of the above
B. delivery of the second twin (If the second twin is breech)
C. delivery of the transverse lie with full dilatation and effacement.
D. delivery of the first twin
81. In one of your CS, you noted Bandl ring, what is your best hysterotomy option?
A. J incision
B. high transverse CS
C. Classical CS
D. Low Transverse Cesarean Section

82. A RESIDENT ATTEMPTED OUTLET FORCEPS DELIVERY ON HR BUT TO NO AVAIL


HENCE PROCEEDED TO CESAREAN SECTION.WHICH OF THE FOLLOWING SHOULD BE
PART OF THE DIAGNOSIS AND INDICATION?
A. INCOMPLETE DELIVERY
B. FAILED FORCEPS
C. FAILED DELIVERY
D. OBSTRUCTED DELIVERY

83. Vacuum extraction has advantages over forceps, except which of the following;
A. Lesser fetal retinal hemorrhage
B. Can be applied in higher station
C. Less maternal laceration
D. It is easier to apply

84. OP, A POST CS PATIENT FOR BREECH PRIMIPARA, ASKED WHEN SHE COULD HAVE
HER SOLID FOOD INTAKE IS AFTER:
A. 6 HOURS
B. 8 HOURS
C. 24 HOURS
D. 10 HOURS

85. TO AVOID BLADDER INJURY DURING CS, WHICH OF THE FOLLOWING SHOULD BE
DONE PRE-OPERATIVELY?
A. GIVE ANATACID
B. 8 HOUR NPO
C. RIGHT/LEFT SIDE WEDGE ON EITHER HIPS
D. INSERT IFC

86. What is the type of forceps is recommended to be used in the delivery of the
aftercoming head?
A. laufe
B. piper
C. Kielland’s
D. Simpson’s
E. tucker-mcklean
F. Option 1

87. PROPER USE OF INDUCTION AGENTS


Option 1

88. When is the suture removal of BB, an obese who underwent CS?
A. 14th post op day
B. 4th post op day
C. 30th post op day
D. 7th to 10th post op day

89. FORCEPS IDEAL FOR ROUNDED FETAL HEAD:


A. SIMPSON or ELLIOT
B. KIELLAND
C. SIMPSON
D. PIPER
E. TUCKER McLANE

90. What is the most favorable prognostic factor when considering trial of labor (TOL) to
have successful vaginal delivery after cesarean (VBAC)?
A. Prior incision type
B. Number of prior CS incisions
C. Prior vaginal delivery
D. Prior incision closure
E. Prior uterine rupture

91. Damage to this nerve can result from prolonged lithotomy position (laying on your
back with your knees bent and legs flexed at your hips while raised in stirrups).
A. Common peroneal nerve
B. Femoral nerve
C. Lumbosacral plexus
D. Pudendal nerve

92. IDENTIFY
Sklar elliot ob forcep

93. TF, FULL TERM IN LABOR ON OXYTOCIN DRIP SUDDENLY COMPLAINED OF SEVERE
HYPOGASTRIC PAIN.UTERINE RUPTURE WAS A CONSIDERATION.WHAT IS THE BEST
ABDOMINAL INCISION FOR THE CASE?
A. MIDLINE VERTICAL INCISION
B. JOEL-COHEN INCISION
C. PFANNENSTIEL INCISION
D. PARAMEDIAN INCISION

94. WHEN SHOULD ONE ABANDON THE VACUUM OR FORCEPS PROCEDURE?


A. PRESENTING PART IS MENTUM POSTERIOR
B. AFTER 3 POP OFFS IN VACUUM EXTRACTION
C. IF THE LOCKS OF THE FORCEPS DO NOT ARTICULATE WELL
D. ALL OF THE CHOICES
E. IF MORE THAN 90 DEGREES IN OUTLET FORCEPS ROATTION IS REQUIRED

Question 95 to 98:
SHOULDER DYSTOCIA DELIVERY MANEUVERS

95. INTERNAL ROTATION - rubin


96. all fours position - gaskin
97. flexing and abducting the maternal hips - mc roberts
98. Cephalic replacement - zavanelli

Question 99 to 101:
PROPER TECHNIQUE

99. increments of 0.2 kg/cm2 every 2 minutes until a total negative pressure of 0.8 kg/cm2 VACUUM
EXTRACTION
100. TRACTION ALONG WITH UTERINE CONTRACTION OUTLET FORCEPS EXTRACTION
101. INCISION AT 45 DEGREES FROM MIDLINE RMLE
A. RMLE
B. VACUUM EXTRACTION
C. OUTLET FORCEPS EXTRACTION

102. The indications and prerequisites for vaginal vacuum delivery are the same as that
of forceps delivery, but vacuum extraction is reserved for fetuses that attain a gestational
age of at least ___ weeks.
A. 28
B. 30
C. 32
D. 34

103. GF, who is going to undergo elective CS was instructed to have NPO prior to
scheduled CS.Ideally, how many hours it must be?
A. 12 hours
B. 6 hours
C. 8 hours
D. 24 hours

104. WHO AMONG THE FOLLOWING IS APPROPRIATE FOR OUTLET FORCEPS


EXTRACTION?
A. COMPOUND PRESENTATION, STATION +#, CERVIX FULLY DILATED AND EFFACED,
RUPTURED BAG OF WATER
B. CEPHALIC, STATION +#, CERVIX FULLY DILATED AND EFFACED, RUPTURED BAG OF WATER
C. MENTUM, STATION + 3, CERVIX FULLY DILATED AND EFFACED, RUPTURED BAG OF WATER
D. CEPHALIC WITH CORD PROLAPSE, STATION +#, CERVIX FULLY DILATED AND EFFACED,
RUPTURED BAG OF WATER
E. BREECH, STATION + 3, CERVIX FULLY DILATED AND EFFACED, RUPTURED BAG OF WATER

105. What is the most important factor when considering trial of labor (TOL) to have
successful vaginal delivery after cesarean (VBAC)?
A. Prior uterine rupture
B. Prior incision closure
C. Prior incision type
D. Prior vaginal delivery
E. Number of prior CS incisions

106. What is the artificial caput formed in vacuum extraction delivery?


A. Cephalhematoma
B. Chignon
C. Caput medusa
D. Molded edema

107. Forceps used for moulded fetal head:


A. SIMPSON
B. PIPER
C. SIMPSON or ELLIOT
D. KIELLAND
E. TUCKER McLANE

108. WHICH OF THE FOLLOWING PROVED TO BE PREVENTIVE OF INFECTION?


A. ENEMA
B. ANTIBIOTIC PROPHYLAXIS
C. ABDOMINOPERINEAL SHAVING
D. PERITONEAL CAVITY WASHING

109. WHO HAS THE HIGHEST CHANCE OF WOUND DEHISCENCE?


A. GR WITH MIDLINE VERTICAL INCISION
B. TF WITH JOEL-COHEN INCISION
C. HA WITH PARAMEDIAN INCISION
D. DD WITH PFANNENSTIEL INCISION
110. In delivering a breech baby who fails to rotate his/her back anteriorly, the best way
to deliver the aftercoming head is by
A. Forceps delivery using the Tucker-McLane forceps
B. Modified Ritgen maneuver
C. Mauriceau maneuver
D. Modified Prague maneuver

111. WHICH OF THE FOLLOWING IS TRUE ABOUT BREECH DELIVERY?


A. EXTERNAL CEPHALIC VERSION SHOULD BE OFFERED TO ALL UNCOMPLICATED BREECH
PRESENTATION AFTER 37 WEEKS AOG
B. ALL TERM BREECHES SHOULD BE DELIVERED BY CS
C. PRESENT STUDIES RECOMMEND THE ELECTIVE CS FOR PRETERM BREECHES
D. ALL OF THE ABOVE

112. The attending resident was exhausted in delivering the aftercoming head, hence you
decided to insert her middle finger into the fetal mouth instead and the second and fourth
fingers are placed on the malar bones just the same.In the OR technique it was described
as such.The consultant asked you what kind of technique was that., the best response
would be?
A. Mauriceu’s maneuver
B. Mauriceau-Smellie-Veit maneuver
C. Wigand-Martin-Winckel maneuver
D. Burns-Marshall maneuver

113. AN OBSTETRICIAN CALLED FOR FORCEPS DELIVERY TO DQ, A G4P3 WITH 3


PREVIOUS VAGINAL DELIVERIES, FOR MATERNAL EXHAUSTION.HOWEVER, THE
PROPERLY APPLIED FORCEPS DO NOT ARTICULATE WELL, WHAT SHOULD BE DONE
NEXT?
A. ROTATE THE FORCEPS
B. APPLY TRACTION TO THE FORCEPS
C. APPLY MORE PRESSURE TO LOCK THE FORCEPS
D. REAPPLY THE FORCEPS
E. DO CS
LONG EXAMINATION -3
(2021 - 2022)
SURE
UNSURE

1. Chemotherapeutic drugs that should be given with extravasation precaution to avoid severe
pain, swelling and necrosis
A. Methotrexate
B. Oncovin
C. Actinomycin
D. Cyclophosphamide

2. This is the type of multifetal pregnancy that results cleavage after the 14th day:
A. Conjoined Twin
B. Monozygotic Twin
C. Multiple Order Twin
D. Dizygotic Twin

3. The cornerstone of therapy in HELLP syndrome is:


A. Anti-hypertensives
B. Blood transfusion
C. MgSO4
D. Delivery

4. Which of the following sentences is true regarding monitoring of patients with preeclampsia
with severe features?
A. Symptoms and vital signs monitored every hour
B. CBC with platelet count every day in asymptomatic patients
C. Doppler studies weekly if with fetal growth restriction (q 2 weeks)
D. BPS weekly (x2 weekly)

5. Which of the following is best for fetal growth restriction prevention?


A. Fetal growth restriction begins as soon as detected
B. In patients with previous babies with fetal growth restriction, Doppler studies are
recommended
C. Accurate dating ideally during early pregnancy
D. Low dose aspirin prevents recurrence of fetal growth restriction

6. The MAP or Mean Arterial Pressure is defined as:


A. Diastolic BP/ Systolic BP
B. Systolic BP x2/ Diastolic BP
C. Diastolic BP + Systolic BP /2
D. Systolic BP + 2x Diastolic BP /3

7. PRENATAL CERVICAL EVALUATION IN ASYMPTOMATIC WOMEN PREVENTS


PRETERM LABOR.
A. True
B. False

8. The hereditary tendency for twin pregnancy may be due to which of the following?
A. Low Androgen levels
B. High Progesterone levels
C. Low Estrogen levels
D. High FSH level

9. Chemotherapeutic drugs associated with secondary tumors such as leukemia, malignant


melanoma and breast cancer
A. Cyclophosphamide
B. Actinomycin
C. Cisplatin
D. Etoposide

10. p 57 kip 2 is an immunostaining that will confirm the diagnosis of partial hydatidiform mole
and hydropic placenta from complete mole. That is…
A. Positive result in complete hydatidiform mole
B. Maternally derived but paternally expressed
C. Paternally derived but maternally expressed
D. Negative result in partial hydatidiform mole and hydropic placenta

11. Which of the following statements about molar pregnancies is TRUE?


A. Older maternal age is not a risk factor for hydatidiform mole
B. The chromosomal make up of a partial or incomplete mole is 46 XX or paternal origin
C. Vaginal bleeding is a common symptom of hydatidiform mole
D. Comprises the malignant group of trophoblastic diseases which is known to invade
locally and metastazise distant sites

12. Primary treatment of choice for gestational trophoblastic neoplasia?


A. chemotherapy
B. radiation therapy
C. surgery
D. curettage

13. The median number of days from uterine evacuation to first normal HCG in patients with
hydatidiform mole:
A. 14 days
B. 28 days
C. 63 days
D. 42 days

14. TRANSVAGINAL ULTRASOUND MEASUREMENT OF CERVICAL LENGTH IS


RECOMMENDED TO THOSE WITH HISTORY OF PRETERM LABOR.
A. True
B. False

15. Prophylactic labor induction in fetal overgrowth is advised….


A. …in non diabetic mother as soon as baby reaches term
B. …if the fetus has an estimated fetal weight of 4500g
C. ..for suspected macrosomia at 38 weeks age of gestation
D. ….to reduce risk of shoulder dystocia

16. Which of the following is NOT a sonographic criteria for a Twin -To-Twin syndrome?
A. Fetus with the same gender
B. Decreased or absent amniotic fluid
C. Placental mass on both placenta
D. Presence of Hydrops fetalis
E. Abnormalities in the fertilization of Complete

17. Hydatidiform mole is due to:


A. Single diploid sperm with haploid ovum
B. Monogynic Diandric Triploidy
C. Exclusively Paternal
D. Normal Haploid Ovum by 2 haploid sperms

18. CERCLAGE COULD BE BENEFICIAL TO THOSE ASYMPTOMATIC SINGLETON


GESTATION WITH BOTH PREVIOUS PRETERM BIRTH & SHORT CERVICAL LENGTH.
A. Yes
B. No
C. Maybe

19. Which statement is true with regards to human chorionic gonadotropin (hCG) levels after
treatment of a molar pregnancy
A. Intervention is required when hCG levels continue to fall
B. if HCG levels arise, then one may have metastatic disease
C. Not needed in monitoring hydatidiform mole patients
D. ANormal levels are usually reached in a week

20. MAINTENANCE TOCOLYSIS AFTER ACUTE THERAPY IS NOT RECOMMENDED.


A. True
B. False

21. In general, no interventions are practiced solely based on fetal age until 42 completed
weeks.
A. Yes
B. No
C. Maybe

22. The probably of having another previously restricted growth fetus (in a mother who had a
baby that was growth restricted)
A. Up to 5%
B. Up to 10%
C. Up to 15%
D. Up to 20%

23. Severe features of preeclampsia is diagnosed in which of the following criteria?


A. Right lower quadrant pain
B. Systolic and diastolic BP of 160 and 110 respectively, occurs twice 4 hours apart,
at bed rest
C. Previous use of anti hypertensive meds prior to pregnancy
D. Presence of pedal edema

24. Normal beta HCG level in a non pregnant state


A. <10 mIU
B. < 5 mIU
C. >5 mIU
D. Any of the choices

25. TRUE of amniotic fluid assessment


A. Decreased amniotic fluid volume at 24 - 34 weeks age of gestation was
significantly associated with malformation
B. There is no association between amniotic fluid volume and fetal growth
C. Amniotic fluid is a good indicator of fetal growth
D. All are TRUE

26. Cesarean section may be advised for non-diabetic mothers with fetus weighing
A. 4500g
B. 5000g
C. 5500g
D. Depends on the mother

27. In growth restricted fetus, when is the best time to give antenatal corticosteroids?
A. Before 30 weeks age of gestation
B. Before 32 weeks age of gestation
C. Before 34 weeks age of gestation
D. Before 36 weeks age of gestation

28. Which condition triggers the FAST PATHWAY of coagulation disorders?


A. Eclampsia
B. Chorioamnionitis
C. Septic Abortion
D. Abruptio placenta

29. Which of the following is an Ultrasonologic finding in Dizygotic Twin?


A. Presence of a thin dividing membrane
B. Opposite sex fetuses
C. Single Placental Site
D. An Empty Lambda sign

30. The only advantage of Dexamethasone use for HELLP syndrome is?
A. Prevents DIC
B. Faster platelet recovery
C. Lessen FFP transfusion
D. Decrease maternal mortality

31. Women with post-term gestations who have unfavorable cervices can best managed by:
A. continuous CTG
B. expectant management even in anxious mother
C. cesarean section
D. labor induction

32. The biometric parameter influenced by shape of the uterus


A. Abdominal circumference
B. Femoral length
C. Placental grade
D. Head circumference

33. How may consolidation course/s is being given in Stage II: high risk patients
A. no consolidation course
B. two consolidation courses
C. one consolidation course
D. three consolidation courses

34. Which of the following structure, show results of notching in early Doppler studies, indicating
the development of Hypertension in pregnancy?
A. Umbilical Artery
B. Uterine Artery
C. Ductus Venosus
D. Middle cerebral artery

35. Importance of first trimester sonography in fetal growth


A. Can predict SGA infants accurately
B. Can identify anomalies significantly
C. Closely estimates age of gestation
D. All are important

36. What do we call a pregnancy that has progressed past the due date?
A. Postdates
B. Overdue
C. Any of the choices
D. Post term

37. This is the leading cause of DIC in pregnancy:


A. Hypertensive complications
B. Infection
C. Acute obstetrical hemorrhage
D. Amniotic fluid embolism

38. The characteristic of the Donor Twin in cases of a Twin to Twin Transfusion syndrome , is:
A. Polycythemia
B. Hydrops Fetalis
C. Growth Restriction
D. Plethoric

39. M.M , 23 year old G1P0 (0010) GTN stage I:10 , s/p suction curettage for hydatidiform mole.
What chemotherapeutic agent would you give?
A. Methotrexate
B. EP-EMA
C. Etoposide-Actinomycin
D. EMACO

40. What is the optimum follow up after chemotherapy ?


A. Option 6
B. monthly for 1 year then every 3 months for 1 year
C. monthly for 6 months, every 2 months for the remaining 6 months, every 3 months for 1
year
D. monthly for 6 months, every 3 months for the remaining 6 months
E. every 3 months for 2 years

41. Histologically, this type of gestational trophoblastic neoplasia has uniform population of
mononucleate trophoblastic cell arranged in nests and cords creating a "geographic" pattern
A. Choriocarcinoma
B. Placental Site Trophoblastic Tumor
C. Invasive Mole
D. Epithelioid Trophoblastic Tumor

42. Which of the following is primarily activated through the Intrinsic Pathway?
A. Toxemia od pregnancy
B. Saline Induced abortion
C. Amniotic fluid embolism
D. Chorioamnionitis
E. Intrauterine fetal death

43. Which of the following is the most commonly used antepartal surveillance amongst post
term pregnancy to avoid its complications?
A. Biophysical profile
B. Cardiotocography
C. Modified BPP
D. Doppler velocimetry studies

44. It is generally not recommended that a pregnancy be allowed to continue past 42 weeks .
A. True
B. False

45. PROGESTERONE COMPOUND PROPHYLAXIS USE IN PRETERM LABOR IS


RECOMENDED TO THOSE EITHER WITH OR WITHOUT PRIOR PRETERM BIRTH BUT
WITH SHORT CERVIX.
A. True
B. False

46. During abnormal platelet activation, which of the following is decreased?


A. Fibrinogen
B. Prostacyclin
C. Prothrombin
D. Thrombin

47. What is the optimum follow up following a diagnosis of hydaditiform mole?


A. every 2 weeks until 3 consecutive normal beta hCG result, then monthly for 6
months
B. every 2 weeks until 3 consecutive normal beta hCG result, then monthy for 12 months
C. monthly until pregnancy test is negative
D. every 2 weeks for 3 months
E. Option 6

48. Which of the following is an exception as it increases the risk of post term pregnancy as it
decreases fetal estrogen production?
A. x- linked placental sulfatase deficiency
B. adrenal hypoplasia
C. fetal anencephaly
D. male sex

49. Which type of trophoblastic cells produces human chorionic gonadotropin and human
placental lactogen
A. Cytotrophoblast
B. Syncytiotrophoblast
C. All of the choices
D. Intermediate trophoblast

50. BY ACOG, UNIVERSAL CERVICAL LENGTH SCREENING IN WOMEN WITHOUT A


PRIOR PRETEM LABOR OR BIRTH IS NOT MANDATORY.
A. True
B. False

51. The ideal age of gestation to deliver a Monoamniotic Twin , is at ____:


A. 34 weeks
B. 40 weeks
C. 36 weeks
D. 38 weeks

52. This pathway involves Thrombin as the activator:


A. Common Pathway
B. Extrinsic pathway
C. Intrinsic Pathway
D. Abnormal Platelet Aggregation

53. The Reticuloendothelial system is responsible for which of the following?


A. Clot formation
B. Adhere to blood vessels
C. Removal of products of coagulation
D. Coagulation factor inactivation

54. Human fetal growth is characterized by sequential patterns and divided into three phases.
This phase is a phase of cellular hyperplasia and hypertrophy
A. 0-16 weeks age of gestation
B. 16 - 32 weeks age of gestation
C. 16 - 28 weeks age of gestation
D. more than 32 weeks age of gestation

55. The mean gestational age of delivery for TRIPLETS is ___.


A. 28 weeks
B. 37 weeks
C. 36 weeks
D. Option 1
E. 33 weeks

56. Getting infected with this DOES NOT cause fetal growth restriction
A. Rubella
B. Cytomegalovirus
C. Syphilis
D. Leptospirosis

57. What is the most common cardiac defect in Twin pregnancy?


A. ASD
B. MS
C. PDA
D. VSD

58. Mrs TE is at her 36th week of gestation. On her check up, her baby was noted to be less
than the 5th percentile of the expected weight. BPP, NST and Doppler studies were normal.
What is the next appropriate step?
A. Give dexamethasone for fetal lung maturity for possible delivery
B. Repeat NST and BPP after 3 days
C. Determine Amniotic fluid volume after 3 days
D. Repeat sonography after 3-4 weeks

59. Most reproducible biometric parameter/ measurement


A. Abdominal circumference
B. Femoral length
C. Biparietal diameter
D. Head circumference

60. Histologically, the hydropic villi are seen within the myometrium or within blood vessels in
the myometrium . What type of gestational trophoblastic neoplasia ?
A. Invasive hydatidiform mole
B. Placental Site Trophoblastic Tumor
C. Epitheliod Type Trophoblastic Tumor
D. Choriocarcinoma

61. CERVICAL LENGTH SCREENING STARTEGY IS CONSIDERED FOR THE PURPOSE


OF TREATMENT WITH VAGINAL PROGESTERONE.
A. True
B. False
62. FETAL FIBRONECTIN SCREENING IS RECOMMENDED AMONG ASYMPTOMATIC
PATIENTS TO DETECT WHO WILL HAVE PRETERM LABOR
A. True
B. False

63. Which of the following leads to fetal overgrowth?


A. Reduced endothelial lipase (High)
B. Diminished maternal glucose (High)
C. Reduced mTORC activity (High)
D. Higher fatty acid binding proteins

64. The safest effective level of Magnesium sulfate in preventing eclampsia is:
A. 7 meq (4-7meq is safe)
B. 5 meq
C. 2 meq
D. 12 meq

65. Parameter that is uniquely linked to fetal growth restriction


A. Abnormal umbilical Doppler velocimetry
B. Decreased flow in the cerebral arteries
C. Increased uterine resistance indices
D. All are unique to growth restriction

66. Which of the following is NOT TRUE regarding the Intrinsic Pathway of activating the
coagulation system?
A. It is a faster process (It is a slow process, Extrinsic is faster)
B. Endotoxins are present
C. There is damage to vessel endothelium
D. Initially activates the Hageman factor

67. Mrs RB is in her 33rd week of gestation and was found to have a very small fetus for the
gestation. Which of the following is an absolute indication for delivery?
A. Amniotic fluid volume of 8cm (AF of 8 is normal)
B. Fetal heart rate of 165 - 170 (NOT persistent yet)
C. Maternal BP 160/100 (Can be managed and delivery can still be delayed)
D. Reverse end diastolic flow

68. If there is evidence of fetal compromise such as fetal bradycardia or oligohydramnios in a


Bishop score less than 5, the best next step is:
A. continuous CTG (If it is persistent fetal compromise then delivery via LTCS)
B. cesarean section
C. labor induction
D. expectant management even in anxious mother
69. In post term, cesarean delivery rate is directly related to station.
A. True
B. False

70. It is a tetra hydrofolic acid derivative that is the antidote used to treat potential side effects of
Methotrexte acid overdose
A. Folinic Acid
B. Granulocyte Colony Stimulating Factor
C. Folic Acid
D. Ranitidine

71. The types and timing of interventions are not unanimous in the management of post term.
A. True
B. False
72. For those with unsure or irregular menses, which of the following is the best basis for age of
gestation?
A. Early UTZ
B. LMP
C. Late UTZ
D. Not sure,I am confused

73. The well accepted risk factor for preeclampsia is:


A. Primparity (Primigravid)
B. Maternal Age
C. Paternity
D. Family History

74. TRANSVAGINAL ULTRASOUND MEASUREMENT OF CERVICAL LENGTH & FETAL


FIBRONECTIN LEVELS AS COMBINED SCREENING TOOL FOR PRETERM LABOR IN LOW
RISK POPULATION IS NOT RECOMMENDED.
A. True
B. False

75. Human fetal growth is characterized by sequential patterns and divided into three phases.
This phase is a phase of hyperplasia
A. 0-16 weeks age of gestation
B. 16 - 32 weeks age of gestation (Cellular Hyperplasia Phase)
C. 16 - 28 weeks age of gestation
D. more than 32 weeks age of gestation (Cellular Hypotrophy)

76. Obstetrical dilemma postulates that ________


A. Our ability to restrict our growth is pathological
B. The ability to walk upright needs a narrow pelvis
C. A narrow pelvis compliments a large head
D. ALL are TRUE

77. This incorporates pregnancies of varying risks, along with the expected outcomes - both
normal and abnormal
A. Population reference birthweight
B. Standard reference birthweight
C. Obstetrical estimate birthweight
D. Mean population birthweight

78. Labor induction in no cervical dilatation leads to 2x cesarean delivery rate for dystocia.
A. True
B. False

79. Post Term is already considered in this age of gestation in a twin pregnancy :
A. 37 weeks
B. 38 weeks
C. 39 weeks
D. 40 weeks

80. CERCLAGE TO THOSE WITHOUT PRIOR PRETERM BIRTH BUT WITH SHORT CERVIX
HAS NO ADVANTAGE.
A. True
B. False

81. It is an antimetabolite cell cycle specific chemotherapeutic drug


A. Vincristine
B. Actinomycin
C. Methotrexate
D. etoposide

82. All are TRUE in Choriocarcinoma EXCEPT:


A. The classic histologic pattern is biphasic
B. The diagnosis cannot be made without demonstrating hydrophic villi
C. The trophoblasts are usually pleomorphic with varying degrees of cytologic atypia
D. A well demarcated hemorrhagic and necrotic mass lesion is seen grossly

83. Most variable biometric measurement


A. Abdominal circumference
B. Placental grade
C. Femoral length
D. Biparietal diameter

84. PRETERM LABOR IS INFLUENCED BY PARITY ALONE


A. True
B. False

85. The most common cause of Disseminated Intravascular Coagulation, is:


A. Hypertension
B. Infection
C. Hemorrhage
D. Malignancy

86. Management of Multifetal Pregnancies highly depend on which of the following?


A. Option 6
B. Chorionicity
C. Cervical length
D. Zygosity
E. Parity of the mother

87. XY 23 year old, G1P0 (0010) GTD s/p suction curettage for hydatidiform mole came for
follow up check up and asked about her future pregnancy. What will you advised to your
patient?
A. She should not get pregnant for 1 year after
B. She should not get pregnant at all
C. She should not get pregnant for 2 years
D. She should not get pregnant at least 6 months

88. An adverse outcome unique to fetal overgrowth


A. Shoulder dystocia
B. Perineal laceration
C. Postpartum hemorrhage

89. All are unique to fetal overgrowth


Prognostic Factor in Staging and Scoring GTN, EXCEPT:
A. antecedent pregnancy
B. size of metastasis
C. prior chemotherapy
D. Blood group

90. Sweeping or stripping of the membranes at 38 to 40 weeks is recommended.


A. True
B. False

91. BC 25 year old, G2P1 (1011) GTN III: 8 in remission, s/p EMACO chemotherapy 5 cycles
s/p suction curettage for hydatidiform mole came for follow up check up and asked about her
future pregnancy. What will you advised to your patient?
A. She should not get pregnant for 1 year after her chemotherapy
B. She should not get pregnant at least 6 months after her chemotherapy
C. She should not get pregnant for 2 years after her chemotherapy
D. She should not get pregnant at all

92. All are TRUE in Partial Hydatdiform mole EXCEPT:


A. Scalloping of the villous outline due to the infolding of the trophoblastic layer
B. Centrally located acellular spaces are filled with fluid
C. Stromal trophoblastic tumor is usually present
D. Small blood cells are usually seen within the villous stroma

93. ADVERSE NEONATAL OUTCOME IS STRONGLY RELATED TO AGE OF GESTATION


AT DELIVERY.
A. True
B. False

94. The first line drug in controlling acute convulsions in eclampsia would be:
A. Nicardipine Drip
B. Diazepam
C. Phenytoin
D. MgSO4

95. BETAMETHASONE USE MAY RESULT TO NEONATAL HYPOGLYCEMIA.


A. True
B. False

96. Mechanism of Action of Etoposide


A. it form interstrand and intrastrand links with DNA to cross link proteins with DNA
B. interacts with topoisomerase II thereby producing single strand breaks in the DNA
C. it binds with tubulin and block the ability of the protein to polymerize into microtubules

97. This type of gestational trophoblastic neoplasia form an “infiltrating nest and sheets
A. ETT
B. Invasive Mole
C. Choriocarcinoma
D. PSTT

98. Which of the following is TRUE regarding management of DIC in Pregnancy?


A. Laboratory tests are specific and routinely requested
B. HELLP syndrome includes activation of the extrinsic pathway
C. Antifibrinolytics can be used in the management all the time
D. There is a case-to-case management scenario

99. The primary oral medication for chronic hypertension in pregnancy is?
A. Labetalol
B. Methyldopa
C. Nifedipine
D. MgSO4

100. AMBULATORY UTERINE MONITORING WITH EXTERNAL TOCODYNOMETER


PREVENTS PRETERM BIRTH.
A. True
B. False
LONG EXAMINATION - 4
(2021 - 2022)
SURE
UNSURE

1. Usually associated with hashimoto D. All of the above


or Addison disease E. None of the above
A. Lymphocytic hypophysitis
B. Prolactinoma 6. Which is NOT transferable or does NOT
C. Diabetes insipidus cross the placenta
D. Acromegaly A. T4
B. All crosses the placenta
2. Which anti Koch’s drug is not given C. TSH
during the maintenance phase D. T3
A. Pyrazinamide
B. Ethambutol 7. Thyroid stimulating immunoglobulins are
C. Rifampicin usually associated with which entity listed
D. Isoniazid below?
A. Graves disease
3. What is the most common cause of heart B. Thyroid peroxidase
failure in pregnancy? C. Fetal micro-chimerism
A. Chronic hypertension D. Thyroiditis
B. Severe preeclampsia E. None of the above
C. Chronic hypertension with
superimposed preeclampsia 8. A unique feature of asthma
D. Idiopathic A. Reversible airway obstruction
B. Bronchial smooth muscle contraction
4. What happens to vital capacity in C. A chronic inflammatory airway
pregnancy? syndrome
A. Increases by 20% D. Has a major hereditary component
B. Increases by 30%
C. Decreases by 45% 9. Mrs ED, a 28 y/o G3P2 (2002) at 32
D. Increases by 40% weeks AOG, consulted at the ER for
E. Decreases by 25% dyspnea. She has a BP of 150/90 mmHg,
temperature of 37C and a respiratory rate of
5.A patient with a term gestation and a 24breaths/minute. Which of the following
history of critical aortic stenosis presents in tests might be helpful?
active labor. Which of the following A. Hematocrit
management strategies is advisable while B. None of the above
caring for her in her labor? C. Thyroid function tests
A. Generous fluid administration D. All of the above
B. Slow-dose epidural anesthesia E. Chest xray
C. Assisted second stage delivery
10. The standard screening test for 15. Ancillary test that gives accurate
pheochromocytoma diagnosis of most heart diseases during
A. Dexamethasone suppression test pregnancy.
B. Elevated 24hr urine free cortisol A. Echocardiography
excretion B. Chest Radiograph
C. Aldosterone-renin ratio C. Arterial blood gas
D. 24 hr urine collection for D. Electrocardiogram
cathecolamines, metanephrines,
or vanillylmandelic acid 16. A 32-year old G2P1 at 17 weeks
gestation reports a history of Tetralogy of
11. TSH levels are monitored at _____ Fallot. She appears comfortable on the
intervals exam table, however, she reports significant
A. 8 - 10 week palpitations and dyspnea when climbing the
B. 2-4 week two flights of stairs to her apartment. Which
C. 6-8 week New York Association class describes her
D. 4 - 6 week functional disability?
A. Class I
12. To compensate for the increased B. Class IV
myocardial consumption, which antenatal C. Class III
change is most important D. Class II
A. Decreased peripheral resistance
B. Increased blood volume 17. Important factor in precipitating cardiac
C. Increased thrombin levels failure
D. Decreased anticoagulant activity A. Blood loss
B. Septic shock
13. Which is TRUE about chest radiography C. Hypertension
during pregnancy? D. Superimposed preeclampsia
A. Lead apron shield is optional
B. The heart silhouette is normally 18. What associated condition is at high risk
larger for developing in a patient who has
C. Fetal radiation exposure is unrepaired ventricular septal defect?
substantial A. Left thrombus
D. Gross cardiomegaly is difficult to B. Bacterial endocarditis
diagnose C. None of the above
D. Embolic stroke
14. Thyrotoxicosis can cause the following E. Venous thromboembolism
EXCEPT
A. ALL are possible 19. Patients with auto-antibodies to TPO
B. Miscariage and thyroglobulin are usually diagnosed
C. Preterm birth with
D. Congenital anomaly A. Subclinical hypothyroidism
B. Isolated maternal Hypothyroxemia
C. Subclinical hyperthyroidism
D. Hyperthyroidism
E. Euthyroid autoimmune thyroid what is the approximate risk of suffering
disease rejection episode during pregnancy?
A. 10%
20. Characterized by a low serum TSH level B. 30%
with normal thyroxine hormone levels C. 20%
A. Subclinical hypothyroidism D. 40%
B. Subclinical hyperthyroidism E. 50%
C. Euthyroid autoimmune thyroid
disease 25. Which of the following is INCREASED?
D. Isolated maternal Hypothyroxemia A. Functional residual capacity
E. Hyperthyroidism B. Minute ventilation
C. Chest wall compliance
21. Which of the following is used for D. Residual volume
treating asthma acutely?
A. Cromolyn 26. Which of the following is the best
B. Terbutaline measure of asthma severity?
C. None of the above A. Chest xray
D. Montelukast B. FEV1
E. Zileuton C. Arterial blood gas
D. Oxygen saturation
22. Treatment of moderate persistent E. Echocardiography
asthma includes low-dose inhaled
corticosteroids and which of the following? 27. In a pregnant patient with mitral
A. Short-acting beta agonist stenosis, which of the following variables is
B. Azithromycin most predictive of a high risk for cardiac
C. High dose inhaled corticosteroids complications during pregnancy?
D. Oral corticosteroids A. Degree of left-sided obstruction
E. Long acting beta agonist B. Functional capacity
C. None of the above
23. A 33 y/o multigravid consulted at the ER D. Ejection fraction
with physical findings of mild exophthalmos,
sweaty skin, and rales at the lung bases. 28. Concerning thyroid storm and heart
Which of the following test will give you the failure, which of the following are true?
most specific result indicating the disease A. It is a dilated cardiomyopathy
process? B. Pulmonary hypertension is common
A. ECG C. All of the above
B. All of the above D. None of the above
C. Thyroid function tests E. Decompensation can be precipitated
D. Chest xray with maternal anemia
E. Hematocrit
29. A symptom or sign that maybe normal
24. In a pregnant woman who has during pregnancy….
previously undergone heart transplantation, A. Orthopnea
B. Chest pain
C. Exercise intolerance B. Euthyroid autoimmune thyroid
D. Persistent split S2 disease
C. Subclinical hyperthyroidism
30. GOALS of bronchial asthma D. Subclinical hypothyroidism
management E. Isolated maternal Hypothyroxemia
A. Including long acting beta 2 agonists
to control exacerbations 35. Which of the following conditions usually
B. Decrease exacerbations improves during pregnancy due to a
C. Decrease use of short acting beta pregnancy induced hypervolemia
2 agonists A. Mitral valve prolapse
D. Maintain normal lung function B. Aortic stenosis
C. None of the above
31. Maternal thyroxine is transferred to the D. All of the above
fetus… E. Mitral stenosis
A. Starting at second trimester
B. Throughout pregnancy 36. During pregnancy, TSH receptors are
C. After the fetal thyroid gland weakly cross stimulated by ______ ?
developed A. B-HCG
D. Starting third trimester B. LH
C. FSH
32. Which of the following ins not a hallmark D. AFP
of asthma? E. GnRH
A. Vascular congestion
B. Mucosal edema 37. This pituitary disorder will most benefit
C. Tenacious mucus from transsphenoidal adenoma resection
D. Reversible obstruction A. Lymphocytic hypophysitis
E. Bronchial smooth muscle B. Acromegaly
relaxation C. Prolactinoma
D. Diabetes insipidus
33. The World Health Organization identifies
which of the following conditions as 38. This pituitary disorder responds well
prohibitive in pregnancy? PULMONARY with dopamine antagonist
ARTERIAL HYPERTENSION A. Acromegaly
A. Heart transplantation B. Diabetes insipidus
B. Cyanotic heart disease C. Prolactinoma
C. Previous myocardial infarction D. Lymphocytic hypophysitis
D. Systemic ventricular dysfunction with
left ventricular ejection fraction of 39. Which of the following best describes
35% the remodeling of the heart that occurs in
normal pregnancy?
34. Pulmonary artery hypertension A. Eccentric left ventricular mass
Characterized by an elevated serum TSH expansion with spherical
level with normal thyroxine hormone levels remodeling
A. Hyperthyroidism
B. Concentric left ventricular mass There is twofold increase in major fetal
expansion with longitudinal malformation
remodeling Has higher hepatotoxicity compared to
C. Concentric left ventricular mass methimazole
expansion with spherical remodeling Has less incidence of transient
D. None of the above leukopenia
E. Eccentric left ventricular mass
expansion with longitudinal 45. If a woman underwent thyroid ablation
remodeling with therapeutic radioactive iodine, when is
she allowed to get pregnant?
40. Hemodynamic changes that has the A. After 3 months
most impact and biggest change during B. After 3 weeks
pregnancy. C. After 6 months
A. Left ventricular stroke work index D. After 6 weeks
B. Heart rate
C. Peripheral vascular resistance 46. A symptom or sign that maybe normal
D. Cardiac output during pregnancy….
A. Diastolic murmur
41. Therapy is administration of vasopressin B. Accentuated respiratory effort
A. Lymphocytic hypophysitis C. Cyanosis
B. Acromegaly D. Syncope
C. Diabetes insipidus
D. Prolactinoma 47. The following endocrinopathy may
cause hypertension in patients except:
42. The fetal response to maternal A. Sheehan syndrome
hypoxemia EXCEPT: B. Acromegaly
A. decreased umbilical blood flow C. Cushing’s disease
B. decreased pulmonary vascular D. Aldosteronism
resistance E. Pheochromocytoma
C. decreased cardiac output
D. alkalosis 48. What happens to the residual volume in
E. increased systemic vascular pregnancy?
resistance A. Decreases by 20%
B. Increases by 20%
43. Fetal exposure to excessive maternal C. Increases by 30%
thyroxine can result to the following D. Decreases by 30%
EXCEPT E. No change
A. ALL are possible
B. Goitrous thyrotoxicosis 49. Lung capacity that is decreased
C. Non-goitrous thyrotoxicosis A. Vital vapacity
D. Non-goitrous hypothyroidism B. Expirations reserve volume
C. Tidal volume
44. TRUE of PTU compared to methimazole D. Inspiratory capacity
Best given during second trimester
50. True of ventricular function in A. Increases by 20%
pregnancy. B. Increases by 25%
A. End systolic and end diastolic C. Increases by 40%
dimensions decrease D. Decreases by 45%
B. There is no change in the septal E. Decreases by 25%
thickness or ejection fraction
C. There is substantive remodeling 56. Occurs in patients with low free serum
characterized by concentric T4 but with normal range TSH
expansion averaging 30 - 35% A. Euthyroid autoimmune thyroid
D. ALL are true disease
B. Subclinical hyperthyroidism
51. Which of the following drugs does NOT C. Hyperthyroidism
inhibit conversion of T4 to T3 D. Isolated maternal Hypothyroxemia
A. Iodine solution E. Subclinical hypothyroidism
B. PTU
C. Hydrocort 57. Which of the following does not
D. Methimazole contribute to the 40% increase in cardiac
output seen during normal pregnancy?
52. Helpful test for Cushing Syndrome A. Decreased systemic vascular
A. Elevated 24hr urine free cortisol resistance
excretion B. Increased stroke volume
B. 24 hr urine collection for C. Increased heart rate
cathecolamines, metanephrines, or D. Increased left ventricular
vanillylmandelic acid contractility
C. Aldosterone-renin ratio E. All of the above contribute to
D. Dexamethasone suppression test increased cardiac output in
pregnancy
53. In developing countries, what is the
most common cause of aortic stenosis? 58. Which of the following is among the four
A. Infectious endocarditis most common maternal complications that
B. Bicuspid aortic valve occur in women with Cushing syndrome?
C. Idiopathic A. None of the above
D. Rheumatic heart disease B. Psychiatric disorders
E. None of the above C. Cardiac failure
D. All of the above
54. MOst common cause of thyrotoxicocis in E. Diabetes
pregnancy
A. Thyroiditis 59. Which of the following medications is
B. Graves disease contraindicated in pregnant patients with
C. Hashimoto’s disease asthma?
D. GTD A. None of the above
B. Prostaglandin E1
55. What effect does progesterone have on C. Oxytocin
tidal volume? D. Prostaglandin E2
E. Prostaglandin F2A C. Tetraploid
D. All of the choices
60. This disease is an autoimmune pituitary
disorder characterized by infiltration and 65. Which of the environmental factors may
parenchyma destruction of the gland cause neurodevelopmental delays in
A. Diabetes insipidus children?
B. Acromegaly A. EXPOSURE TO COMPUTERS
C. Lymphocytic hypophysitis B. EXPOSURE TO SAUNAS/ BATH
D. Prolactinoma TUBS
C. CONSUMPTION OF MERCURY
61. A maternal adaptation that contributes CONTAINING FISH
to the hypercoagulable state: D. Option 6
A. .Increased RBC production E. EXPOSURE TO HYPOTHERMIA
B. Increased plasma volume
C. Decreased Protein C and S 66. What is the most common sex
activity chromosome abnormality?
D. Decreased peripheral vascular A. 47,XYY and 47,XXX
resistance B. Klinefelter’s syndrome
C. Cri-du-chat syndrome
62. What describes some genes that are D. Monosomy X syndrome
inherited but not expressed, depending on
whether they are inherited from the mother 67. JK, 35 year old newly wed and anxious
or father? of getting pregnant with congenital anomaly
A. Consanguinity because of her age, which of the following is
B. Mitochondrial Inheritance not linked with her advance maternal age?
C. Imprinting A. Turner’s syndrome
D. Anticipation B. Down’s syndrome
C. Patau’s syndrome
63. True about Standard Nomenclature in D. Edward’s syndrome
Genetics EXCEPT:
A. description of any structural 68. Which is used in the postnatal setting to
variation is listed also but not the evaluate suspected genetic syndromes and
specific abnormalities intellectual disability?
B. chromosomes correspond to the A. Whole genome sequencing (WGS)
number of centromeres /Whole exome sequencing (WES)
C. total number of chromosomes is B. chromosomal microarray analysis
listed first followed by the sex (CMA)
chromosomes C. cytogenetic analysis
D. none of the choices (karyotyping)
D. fluorescence in situ hybridization
64. Which of the following presents as molar (FISH)
pregnancy?
A. Digynic triploid
B. Diandric triploid
69. What is the most common inherited form C. Mosaicism
of intellectual disability, autism or “autistic D. Monosomy
like” behavior?
A. Monosomy X 75. TRUE about X-linked inheritance:
B. 47XYY A. A female with an X-linked disease
C. 47XXY cannot have a affected sons
D. Fragile X Syndrome (Usually Sons are affected)
B. Women with an X-linked recessive
70. If structural abnormalities are identified, gene are generally unaffected but
what is the first step to offer? may manifest due to lyonization
A. cytogenetic analysis (karyotyping) (Lyonozation occurs in Male)
B. chromosomal microarray analysis C. X-linked dominant disorders mainly
(CMA) affect females and tend to be lethal
C. Whole genome sequencing (WGS) (Males)
/Whole exome sequencing (WES) D. Most X-linked diseases are
NS dominant.
D. fluorescence in situ hybridization E. All are true
(FISH)
76. What is the most common aneuploidy?
71. WHAT IS THE MOST COMMON FETAL A. Trisomy 1
CONGENITAL ANOMALY? B. Trisomy 13
A. G6PD deficiency C. Trisomy 18
B. neural tube defects D. Trisomy 21
C. Down syndrome
D. heart defects 77. Which of the following detects
microduplications and microdeletions as
72. What is the most common birth defect? small as 50 to 100 kilobases?
A. Musculoskeletal defects A. cytogenetic analysis (karyotyping)
B. Metabolic defects B. chromosomal microarray analysis
C. Cardiac Defects (CMA)
D. Neural-Tube Defects C. Whole genome sequencing (WGS)
/Whole exome sequencing (WES)
73. What is the most common cause of D. fluorescence in situ hybridization
Trisomy 21—Down Syndrome ? (FISH)
A. isochromosome
B. nondisjunction 78. What is the most easily recognized
C. robertsonian translocation numerical chromosomal abnormalities?
D. Mosaicism A. Triploidy
B. Aneuploidy
74. Which of the following Turner’s C. Monosomy
syndrome with Y chromosome needs D. Polyploidy
prophylactic bilateral gonadectomy ?
A. Aneuploidy
B. All of the choices
79. It explains the why some diseases suggests a specific aneuploidy is identified,
appear to follow more than one type of such as an endocardial cushion defect
inheritance: (trisomy 21) or alobar holoprosencephaly
A. Locus heterogeneity (trisomy 13), karyotyping, what test should
B. Phenotypic heterogeneity be done or offered?
C. Genetic heterogeneity A. chromosomal microarray analysis
D. Allelic heterogeneity (CMA)
B. Whole genome sequencing (WGS)
80. Which of the following is highly lethal? /Whole exome sequencing (WES)
A. Turner’s syndrome C. fluorescence in situ hybridization
B. Down’s syndrome (FISH)
C. Edward’s syndrome D. cytogenetic analysis (karyotyping)
D. Patau’s syndrome (Most die in the
first months and other by 1 year) 85. Cell-Free DNA importance in fetal sex
determination is important in:
81. Most of the Genetic disease is A. Rh D alloimmunization
commonly recognized among: B. congenital adrenal hyperplasia
A. Age 5 C. early demise of an aneuploid cotwin
B. Age 18 D. Autosomal Recessive Inheritance
C. Day 3 E. fetal autosomal trisomy
D. Newborn
86. WHICH OF THE FOLLOWING
82. Which is used for rapid identification of a INFECTIONS MOST LIKELY CAN CAUSE
specific chromosome abnormality and for CONGENITAL CATARACT?
verification of suspected microdeletion or A. RUBELLA
duplication syndromes? B. HIV
A. Whole genome sequencing (WGS) C. HERPES
/Whole exome sequencing (WES) D. CMV
B. cytogenetic analysis (karyotyping)
C. fluorescence in situ hybridization 87. True about extra X and Y
(FISH) chromosomes:
D. chromosomal microarray analysis A. For both males and females, the
(CMA) IQ score is lower with each
additional X chromosome
83. Inborn errors of metabolism is classified B. Makes one superior
under which of the following mendelian C. They have “ ALPHA” female or male
inheritance ? complex due to aggressiveness
A. X-Linked Inheritance D. All of the choices
B. Y-Linked Inheritance
C. Autosomal Recessive Inheritance 88. Which of the following is seen as
D. Autosomal Dominant Inheritance septated cystic hygromas and or hydrops
fetalis?
84. If a particular anomaly that strongly A. Turner’s syndrome
B. Patau’s syndrome
C. Edward’s syndrome C. Autosomal Dominant Inheritance
D. Down’s syndrome D. Autosomal Recessive Inheritance

89. Increased rates of which complication 94. Which type of mendelian inheritance if
has been linked to binge drinking during only one copy of a gene pair determines the
pregnancy? phenotype?
A. Stillbirth A. Y-Linked Inheritance
B. fetal growth restriction B. X-Linked Inheritance
C. Postpartum depression C. Autosomal Recessive Inheritance
D. Preterm Birth D. Autosomal Dominant Inheritance

90. Which of the following detects numerical 95. What is the most common nonlethal
abnormalities (aneuploidy)? trisomy?
A. chromosomal microarray analysis A. Patau’s syndrome
(CMA) B. Edward’s syndrome
B. Whole genome sequencing (WGS) C. Turner’s syndrome
/Whole exome sequencing (WES) D. Down’s syndrome
C. fluorescence in situ hybridization
(FISH) 96. NOT True about recurrence of
cytogenetic analysis (karyotyping) autosomal trisomy:
A. Recurs in 1 percent
91. For stillbirth evaluation,what should be B. Still maternal age related
the test of choice? C. All of choices
A. cytogenetic analysis (karyotyping) D. Parental chromosomal studies is a
B. chromosomal microarray analysis must
(CMA)
C. Whole genome sequencing (WGS) 97. At what stage would radiation has an “all
/Whole exome sequencing (WES) or none” effect to the embryo?
D. fluorescence in situ hybridization A. 0-2 weeks AOG
(FISH) B. 2-8 weeks AOG
C. 8-15 weeks AOG
92. What explains why some autosomal D. 16-25 weeks AOG
dominant diseases may appear to “skip”
generations? 98. What is the phenomenon in which
A. Advanced Paternal Age disease symptoms seem to be more severe
B. Codominant Genes and to appear at an earlier age in each
C. Expressivity successive generation?
D. Penetrance A. Consanguinity
B. Imprinting
93. Offspring with congenital anomaly due C. Mitochondrial Inheritance
to consanguineous relationship are usually D. Anticipation
transmitted as:______
A. Y-Linked Inheritance 99. Which of the following is compatible with
B. X-Linked Inheritance life, but it is also the most common
aneuploidy in first-trimester losses? Trisomy
16
A. Turner’s syndrome
B. Patau’s syndrome
C. Edward’s syndrome
D. Down’s syndrome

100. It is a process where persistent fetal


cells may engraft in the mother causing
some maternal autoimmune diseases.
A. Imprinting
B. Microchimerism
C. Chimerism
D. Mosaicism

101. CG, 40 years old first timer mother on


her 30 weeks AOG is suspected to be
carrying a fetus with congenital anomalies
and a noted hyperplacentosis.Other than
this, there is a new onset of hypertension.
Which of the following is closely related to
hypertension in pregnancy?
A. Patau’s syndrome
B. Turner’s syndrome
C. Down’s syndrome
D. Edward’s syndrome
LONG EXAMINATION - 5
(2021 - 2022)
SURE
UNSURE

1. A pregnant patient presents with


loose bowel movements for 3 days. 4. An asymptomatic 20 year old G1P0
She is afebrile, has not had blood- at 18 weeks gestation present for
tinged stools, and is able to tolerate her first prenatal visit. A urinalysis
liquids without vomiting. Which of and urine culture are sent as part of
the following drugs is indicated? routine care. the results show >100,
A. Metronidazole 000 gram-negative rods. The
B. All of the choices diagnosis is which of the following?
C. Trimethoprim-sulfamethoxazole A. Cystitis
D. Bismuth subsalicylate B. Pyelonephritis
E. Ciprofloxacin C. Nephrotic syndrome
D. Diverticulitis
2. WHICH OF THE FOLLOWING E. Asymptomatic bacteriuria
CONSIDERED THE MAINSTAY OF
MEDICAL TREATMENT OF 5. FIRST LINE TREATMENT FOR
HYPEREMESIS GRAVIDARUM? GERD IN PREGNANCY IS:
A. PROMETHAZINE A. ANTACIDS
B. METOCLOPRAMIDE (old answer) B. ANTIHISTAMINES
C. DOXYLAMINE+ PYRIDOXINE C. PROTON PUMP INHIBITORS
D. ONDANSETRON D. PHENOTHIAZINES

3. DG, a G3P2 at 39 weeks’ gestation 6. Which of the following is a risk factor


presents in active labor. The patient of PUPPP?
has a history of genital herpes. A. Female fetus
During inspection of the perineum B. Multiparity
and cervix, no lesions are seen. C. African American ethnicity
However, a suspicious, tender lesion D. Multifetal gestation
is seen on the patient’s inner thigh, 3
cm above the knee. Which of the 7. Maternal complication of bowel
following is the best management preparation using polyethylene
plan? glycol electrolyte or sodium
A. Cesarean delivery and oral acyclovir phosphate solutions.
B. Tocolysis and oral acyclovir A. vomiting
C. Occlusive dressing over the B. fetal congenital anomaly
lesion and permit vaginal delivery C. preterm birth
D. Tocolysis and send swab of lesion D. serious maternal dehydration
for PC R assay for H SV
8. Associated with hepatitis C virus C. Chronic renal insufficiency
A. Atopic eruption of pregnancy D. Connective tissue disorders
B. Impetigo Herpetiformis
C. Pemphigoid Gestationis 13. What is the strongest risk factor for
D. Polymorphic eruption of pregnancy hyperemesis gravidarum requiring
E. Intrahepatic cholestasis of hospitalization?
pregnancy A. Twin gestation
B. Hospitalization in a previous
9. A 27 year old G1P0 with a history of pregnancy for hyperemesis
swallowing problems presents with gravidarum
dysphagia, chest pain and vomiting. C. H. Pylori infection
A barium swallow done just prior to D. Female fetus
pregnancy revealed these images.
What is the patient's diagnosis? 14. Which of the following statements
Captionless Image regarding PUPPP is true?
A. Peptic Ulcer Disease A. Patients should be counseled about
B. Intestinal Obstruction the significant scarring that will likely
C. Hiatal Hernia occur (No scar)
D. Achalasia B. It seldom recurs in subsequent
E. Diaphragmatic Hernia pregnancies
C. Lesions originate in the periumbilical
10. To whom should one do CS to avoid area (Within striae but periumbilical
or reduce neonatal transmission of sparing)
infection? D. Symptoms frequently persist for 6 to
A. Chlamydia 9 months postpartum (resolves
B. Hepatitis B within several days following
C. HIV delivery. Rarely upto 2-4 weeks after
D. Gonorrrhea delivery)

11. Among which of the cellular 15. Which of the following is a threshold
reactions in “cytokine storm”most for proteinuria in pregnancy, above
likely causes myocardial which levels are considered
suppression? abnormal?
A. Proteases A. 100 mg/day
B. Interleukin-6 (IL-6) B. 200 mg/day
C. Oxidants C. 350 mg/day
D. Tumor necrosis factor-α (TNF-α) D. 300 mg/day
E. 150mg/day
12. Most common chronic disorder
encountered as a cause of anemia 16. WHICH OF THE FOLLOWING
during pregnancy? COMPLICATIONS IS A VITAMIN
A. Inflammatory bowel disorders (old DEFICIENCY CAUSED BY
answer) HYPEREMESIS GRAVIDARUM?
B. Malignant neoplasms
A. WERNICKE ENCEPHALOPATHY - FOODS, GIVEN MEDICATIONS BUT TO
Thiamine Def NO AVAIL.tHE BEST NEXT STEP WOULD
B. ALL OF THE CHOICES BE:
C. FETAL INTRACRANIAL A. MISOPROSTOL
HEMORRHAGE - NOT INCLUDED B. ANY OF THE CHOICES
D. HYPOPROTHROMBINEMIA - C. FUNDUPLICATION
Vitamin K def D. ENDOSCOPY

17. TRUE ABOUT ANATOMIC RENAL 20. TRUE about ACUTE PULMONARY
CHANGES IN PREGNANCY. EDEMA during pregnancy :
A. there is increase in kidney size due A. noncardiogenic pulmonary edema is
to hypertrophy less common
B. all of the choices B. all of the choices
C. none of the choices C. more than half who develop
D. there dilatation of the ureters and pulmonary edema have some
pelvis due to progesterone degree of sepsis syndrome
E. there is vesicoureteral reflux due to cardiogenic pulmonary edema is more
stasis frequent & precipitated by iatrogenic fluid
overload in hemorrhage resuscitation and
17. FG, a G1P0 at 36 weeks’ gestation at vigorous tocolytic therapy
OPD was seated with a px coughing and
scratching next to her. She was exposed to 21. This dermatoses has a characteristic
chicken pox, and she denies having had it in decreased TH1 response and increased
the past, what should be done? TH2 response, which worsens already
A. Test her for VZV serology and, if present TH2 imbalance in atopic patients
negative, give vaccine and VariZIG A. Polymorphic eruption of pregnancy
B. Skip V ZV serology testing and B. Impetigo Herpetiformis
administer VariZIG C. Atopic eruption of pregnancy
C. Test her for varicella zoster virus D. Intrahepatic cholestasis of
(VZV) serology and give her pregnancy
VariZIG if negative E. Pemphigoid Gestationis
D. Offer her vaccination
22. PX CV, underwent cesarean delivery
18. WHEN IS THE IDEAL SURGICAL became ill and with suspected peritonitis on
INTERVENTION IN SEVERE HIATAL 4th post op day.Which of the following is the
HERNIA? primary consideration?
A. 34 WEEKS AOG A. uterine abscess
B. 12 WEEKS AOG B. uterine gangrene
C. ANYTIME C. bowel perforation
D. 20 WEEKS AOG D. uterine incisional necrosis

19. TP, 12 WEEKS PREGNANT, 23. Which of the following Acute pulmonary
COMPLAINED OF HEARTBURN. SHE edema treatment is TRUE?
WAS ADVISED TO AVOID THE TRIGGER
A. Furosemide is given in 20- to 40-mg 27. What is the most common cause of
intravenous doses but not along with thrombocytopenia in pregnancy?
anti HPN drugs A. Severe preeclampsia
B. Echocardiography help direct B. Gestational thrombocytopenia
further therapy C. Immune thrombocytopenic purpura
C. Immediate delivery via emergency D. Consumptive coagulopathy
cesarean is a definitive treatment (not
an indication) 28. What is the classic morphological
D. A live fetus does not prohibits the evidence of erythrocytes in iron deficiency
use of cardioactive drugs that might anemia?
rapidly lower peripheral resistance A. Microcytic, hypochromic
and in turn severely diminish B. Normocytic, hypochromic
uteroplacental circulation C. Microcytic, normochromic
D. Macrocytic, hypochromic
24. GR, a 40 year old primipara with
preeclampsia suddenly developed difficulty 29. Which of the following statements about
of breathing and on auscultation, bilateral pemphigoid gestationis is true?
crackles was appreciated, which is the most A. It is most commonly seen on the
probable cause of pulmonary edema in the face
case? B. It typically starts in the first trimester
A. Cardiogenic of pregnancy
B. noncardiogenic C. It has been associated with
C. hydrostatic edema preterm birth and fetal growth
D. high pulmonary capillary hydraulic restriction
pressures D. It results from an infection with
E. all of the choices herpes virus

25. Of the following, which is the most 30. To whom should pneumocystis
common cause of nephrotic syndrome pneumonia prophylaxis be given among
A. Minimal Change Disease AIDS diagnosed pregnant women?
B. Focal Segmental A. if there is an AIDS-defining illness,
Glomerulosclerosis particularly oropharyngeal
C. Amyloidosis candidiasis
D. Membranous glomerulonephritis B. those whose CD4+ T lymphocytes
constitute less than 14 percent
26. WHICH OF THE FOLLOWING GIT C. CD4+ T-lymphocyte counts < 200/μL
DISORDER (ILEUS) WILL MOST LIKELY D. all of the choices
DEVELOP AFTER CS?
A. MALLORY-WEISS TEARS 31. A RADIOGRAPHIC BARIUM
B. ACHALASIA SWALLOW FINDINGS OF BIRD BEAK OR
C. BORHAAVE'S SYNDROME ACE OF SPADES IS PATHOGNOMONIC
D. WERNICKE ENCEPHALOPATHY OF:
E. OGILVIE SYNDROME A. PEPTIC ULCER DISEASE
B. ACHALASIA
C. HIATAL HERNIA B. Lesions become smaller but less
D. DIAPHRAGMATIC HERNIA numerous
C. Pregnancy has no effect
32. Which of the following is not a D. Lesions may increase in size and
recommended requisite for renal transplant number
patients who want to attempt pregnancy?
A. Proteinuria <300mg/day <500 36. Sheena, 26 y/o primigravid in her 20th
B. no identifiable pyelocalyceal week of gestation. She developed severe
distention by urography itching with papules and plaques in the face
C. Serum creatinine <2 mg/dL neck and arms. She was known to have
D. No evidence of graft rejection for 6 allergies to seafood and some allergens
months Most probable diagnosis?
A. Impetigo Herpetiformis
33. AR, 23 y/o primigravid in her 34th week B. Polymorphic eruption of pregnancy
of gestation. She developed severe itching C. Pemphigoid Gestationis
with papules and plaques in the abdomen D. Intrahepatic cholestasis of
with extension to the umbilicus. Most pregnancy
probable diagnosis? E. Atopic eruption of pregnancy
A. Polymorphic eruption of pregnancy
B. Atopic eruption of pregnancy 37. During pregnancy, which of the following
C. Impetigo Herpetiformis is decreased?
D. Pemphigoid Gestationis A. sebaceous function
Intrahepatic cholestasis of pregnancy B. melanin production
C. eccrine function
34. TRUE ABOUT RECOMMENDED D. apocrine function
SCREENING OF ASYMPTOMATIC
BACTERIURIA EXCEPT: 38. Which of the following promotes
A. Dipstick culture technique has nephrolititasis?
excellent positive- and negative- A. Low-sodium diet
predictive values B. hydration
B. Susceptibility determination is C. Low-calcium diet
necessary to give the right D. Thiazide diuretics
antimicrobilas.
C. Standard urine cultures may not be 39. In puerperal pelvic
cost effective when the prevalence is infections,hysterectomy is indicated in which
low. of the following condition?
D. Done at the first prenatal visit A. all of the choices
B. septic abortion with suspected
35. Which of the following is true regarding abscess
the effect of pregnancy on the size and C. metritis
number of neurofibromatosis lesions? D. uterine gangrene
A. Lesions become smaller but more
numerous 40. What is the most common reason for
acute kidney injury in obstetrics?
A. Use of loop diuretics 45. A patient in the second trimester
B. Hyperemesis presents with colicky abdominal pain,
C. Acute blood loss nausea, and vomiting. At laparotomy,
D. Ureteral injury at cesarean section surgical findings are similar to those shown
here. What is her likely diagnosis?
41. When should treatment of asymptomatic Captionless Image
bacteriuria be given? A clean-voided A. Intestinal Obstruction
specimen containing__________ B. Ulcerative colitis
A. any of the above C. Crohn Disease
B. only 50,000 organisms/mL D. Perforation
C. more than 100,000 organisms/mL E. Colonic volvulus
D. only 20,000 organisms/mL
46. After discharge from the hospital for
42. Which of the following pregnancy acute pyelonephritis, patients should
complications has been associated with receive oral antimicrobial treatment for how
polycythemia vera? long?
A. Abruptio placenta A. none of the choices
B. Coagulopathy B. 7-14 days
C. Placenta previa C. antimicrobials are not necessary
D. Stillbirth D. 21-28 days
E. 3-5 days
43. A 22 year old G1P0 at 14 weeks
gestation with a history of type 1 diabetes 47. What is the primary treatment for
presents for prenatal care. Her diabetes is Immune Thrombocytopenic Purpura?
poorly controlled and her serum creatinine A. Platelet concentrate
is 3.5mg/dL. While counseling her regarding B. Platelet concentrate, Corticosteroids
the prognosis of this pregnancy, how would C. IVIG
you categorize her renal impairment? D. Corticosteroids, IVIG
A. none of the choices
B. Mild renal insufficiency 48. A 21-year-old female admitted for 3 day
C. Severe renal insufficiency history of diarrhea with severe renal
D. all of the choices involvement and no neurological symptoms
E. Moderate renal insufficiency was diagnosed with Thrombotic
microangiopathy. What is the cornerstone of
44. When is the best time to do ERCP treatment?
during pregnancy? A. Dialysis
A. 3rd trimester to be sure that the B. IVIG
organs are already developed C. Plasma exchange
B. anytime to avoid complications D. Hydration
C. 2nd trimester so that the uterus is
not so big 49. What is true about Zika infection?
D. first trimester so that the uterus is A. partner who lives in or has traveled
small enough to an area with Zika, use condoms
during sex (vaginal, anal, or oral) to
protect against infection or do not 53. Which of the following is an absolute
have sex during the pregnancy. contraindication in relation to ERCP during
B. All of the above pregnancy?
C. The most common sign of Zika A. fluoroscopy/radiation
infection is fever B. anesthetic medications
D. The most common fetal sign of Zika C. supine position
infection is convulsion D. term uterine size

50. Which of the following is true regarding 54. Which of the following is the most
the effect of pregnancy on the size and common pregnancy-specific dermatosis?
number of neurofibromatosis lesions? Cutaneous lupus
A. Lesions become smaller but less Pruritic folliculitis of pregnancy
numerous Eczema in pregnancy
B. Lesions may increase in size and Erythema nodosum
number
C. Lesions become smaller but more 55. Which of the following statements
numerous regarding the physiologic changes of
D. Pregnancy has no effect pregnancy is true?
A. Effective renal plasma flow
51. When managing a pregnant woman on decreases
dialysis, which of the following is an B. all of the choices
important modification? C. none of the choices
A. reduce doses of daily oral D. Glomerular Filtration remains
multivitamins unchanged
B. none of the choices E. Glomeruli become larger because of
C. extend dialysis to 5-6 times weekly an increased number of cells
to avoid hypotension (hyperplasia)
D. add additional bicarbonate and less F. Serum creatinine concentration
calcium dialysate increases
E. all of the choices
F. stop erythropoietin because of 56. Which of the following is associated with
harmful fetal effects Fitz-Hugh-Curtis Syndrome (Perihepatitis)?
A. Candidiasis
52. Which of the following is most diagnostic B. Gonorrhea
for cardiomyopathy in pregnancy? C. C. Trachomatis
A. echocardiography systolic function / D. Trichomoniasis
ejection fraction
B. brain natriuretic peptide (BNP) > 500 57. Which of the following noninvasive
pg/mL imaging techniques is NOT recommended
C. echocardiography diastolic during pregnancy?
dysfunction A. computed tomography (CT)
D. all of the choices B. abdominal sonography
C. magnetic resonance
cholangiopancreatography—MRCP
D. magnetic resonance C. Erythema nodosum
enterography—MRE D. Neurofibromatosis

58. What is the cornerstone of treatment for 63. What is the leading cause of septic
thrombotic thrombocytopenic purpura? shock in pregnancy?
A. Plasmapheresis A. Pyelonephritis
B. Intravenous immunoglobulin B. Pelvic Inflammatory Disease
(IVIG) C. Pneumonia
C. Anticoagulation D. Breast Abscess
D. Platelet transfusion E. Chorioamnionitis

59. This dermatoses has a charateristic 64. WHAT IS THE MOST COMMON
decreased TH1 response and increased URINARY TRACT INFECTION?
TH2 response, which worsens already A. ASYMTOMATIC BACTERIURIA
present TH2 imbalance in atopic patients B. SYMPTOMATIC BACTERIURIA
Polymorphic eruption of pregnancy C. CYSTITIS
A. Intrahepatic cholestasis of D. PYELONEPHRITIS
pregnancy
B. Pemphigoid Gestationis 65. True about mechanical ventilation in
C. Atopic eruption of pregnancy pregnancy :
D. Impetigo Herpetiformis A. Mechanical ventilation with initial
tidal volume >/=6 mL/kg is optimal
60. What is the most common complication B. Early intubation is preferred in the
of parenteral nutrition during pregnancy? gravida if respiratory failure is
A. brachial plexus injury more likely than not or is
B. hemothorax imminent
C. pneumothorax C. Lower levels for Pao2 is required
D. catheter sepsis D. High-frequency oscillation ventilation
(HFOV) is crucial in ARDS
61. Which of the following is the most E. Early stages of pulmonary
common dermatologic disorder, occurring in insufficiency require positive
1 percent of pregnancies? pressure ventilation by nasal
A. Pemphigoid cannula will suffice
B. Atopic eruptions
C. Intrahepatic cholestasis of 66. TRUE ABOUT RENAL FUNCTION
pregnancy DURING PREGNANCY EXCEPT:
D. Pruritic urticarial papules and A. decreased potassium excretion
plaques of pregnancy (PUPPP) B. increased urinary protein excretion
C. decreased BUN and creatinine
62. Which of the following conditions is levels
characterized by facial pustules and D. increased glomerular filtration rate
coalescing draining sinuses? (GFR)
A. Pyogenic granuloma
B. Rosacea fulminans
67. What is the effective therapeutic option B. in pregnancy, prognosis is
for pregnant women diagnosed with ITP dependent on the degree of
with no response to IVIG or corticosteroid? hypertension and renal insufficiency
A. Cytotoxic agents C. heavy proteinuria is the hallmark
B. Splenectomy D. all of the choices
C. Platelet concentrate E. none of the choices
D. all of the choices
71. What is the CDC recommended
68. Which of the following is the preferred treatment for influenza pneumonitis?
agent for treatment of rejection in post A. Oseltamivir
kidney transplant during pregnancy? B. neuraminidase inhibitors
A. Azathioprine C. All of the choices
B. Methylprednisolone D. amantadine or rimantadine
C. Prednisone
D. Calcineurin inhibitor 72. Which of the following is the most
E. Mycophenolate mofetil and sirolimus dreaded reason why UTI must be monitored
and treated during pregnancy?
69. Pretty, a 28-year-old G5P4 presents in A. MAY LEAD TO UROSEPSIS
active labor. She has had no prenatal care, B. Bladder sensations can also be
but she appears to be term. A rapid H IV obscured
test is performed, and results are positive C. urinary stasis, vesicoureteral reflux,
for HIV. Which of the following is the best and diabetes is increased during
management plan for this patient? pregnancy
A. Complete H IV counseling, start D. THERE IS REVERSAL OF T-helper
antiretroviral prophylaxis without cell—Th1/Th2 ratio
delay, and postpone breast
feeding until confirmatory test 73. Which of the following is technology is
results are available indispensable in interrogating cardiac
B. Complete H IV counseling, start anatomy and especially right-ventricular
antiretroviral prophylaxis, and function ?
continue it for 6 months ➔ 2-D echo
C. Send a confirmatory test and start ➔ All of the choices
antiretroviral prophylaxis only if this ➔ CXR
second confirmatory test result is
➔ ECG
positive for H IV
D. Start antiretroviral prophylaxis
74. WHICH OF THE FOLLOWING
immediately and complete H IV
CAUSATIVE AGENTS FOR COMMON
counseling postpartum to avoid
ACUTE DIARRHEAL SYNDROMES DOES
undue anxiety during labor
NOT REQUIRE PHARMACOLOGIC
TREATMENT?
70. Select correct statement regarding
A. ROTAVIRUS
nephrotic syndrome
B. E. COLI (Hemorrhagic)
A. there is increased risk of
C. ALL OF THE CHOICES
thromboembolism
D. STAPHYLOCOCCUS
D. ampicillin + gentamicin +
75. WHICH OF THE FOLLOWING clindamycin +vancomycin+ tetanus
GASTRIC CONDITIONS WOULD toxoid
RESPOND TO TREATMENT OF H.
PYLORI? 79. Why is high dose folic acid
A. PEPTIC ULCER DISEASE supplementation indicated in pregnant
B. ACHALASIA patients with inflammatory bowel disease?
C. DIAPHRAGMATIC HERNIA A. To prevent bowel disease relapses
D. HIATAL HERNIA B. To prevent fetal cardiac defects
C. To counteract the antifolate
76. In oxygen delivery ,it should be actions of sulfasalazine
remembered that: D. To act synergistically with calcium in
A. goal for Paco2 of < 35 - 45 mm Hg the prevention osteoporosis
B. each gram of hemoglobin carries
1.25 mL of oxygen when 90- 80. Px N.K., 26 weeks AOG was brought to
percent saturated ER due to fever, vomiting and hematuria.
C. goals in severe lung injury is to Pertinent PE revealed costovertebral
attain a Pao2 of 90 mm Hg or 60% tenderness. What is the diagnosis?
oxygen saturation A. SYMPTOMATIC BACTERIURIA
D. increasing the arterial Po2 from 100 B. CYSTITIS
to 200 mm Hg results into better C. ASYMTOMATIC BACTERIURIA
oxygenation D. PYELONEPHRITIS

77. WHICH OF THE FOLLOWING IS A 81. What defines acute respiratory distress
PREGNANCY RELATED syndrome (ARDS)
GASTROPROTECTION? A. Pao2 :Fio2 ratio is < 300 coupled
A. ALL OF THE CHOICES with dyspnea, tachypnea, oxygen
B. INCREASED MUCUS desaturation, and radiographic
PRODUCTION pulmonary infiltrates
C. REDUCED GASTRIC ACID B. Acute lung injury and severe
SECRETION permeability pulmonary edema and
D. DECREASED MOTILITY respiratory failure
C. Pao2:Fio2<200, and no evidence of
78. Px FT, a diagnosed case of pelvic heart failure
infection s/p pelvic clean up.However,you D. All of the choices
noted SSI on the 4th post op day.What is
the best antibiotic regimen for the patient? 82. Which of the following is not a common
A. ampicillin + gentamicin cause of reactive thrombocytosis?
B. ampicillin + gentamicin + A. Malignant tumors
clindamycin +vancomycin B. Infection
C. ampicillin + gentamicin + C. Inflammatory disease
clindamycin D. None of the choices
E. Iron deficiency
83. What is the most common cause of 150/100mmgHg, and no fetal heart tones
sepsis syndrome in obstetrics? are seen monographically. She has 4+ urine
A. PROM protein. the patient is diagnosed with severe
B. Pyelonephritis preeclampsia and with fetal demise due to
C. Abortion suspected abruptio placenta. You send a
D. UTI panel of laboratory tests, tart magnesium
sulfate infusion, transfuse 2 units PRBVC
84. Which of the following statements and induce labor. The patient has a prompt
regarding intrahepatic cholestasis of vaginal delivery with an estimated 1L of
pregnancy is true? blood loss. Postpartum, her urine output is
A. It has not been linked to adverse 20ml/hr, HR is 120 bpm, and BP is 90/60.
fetal outcome Which of the following is your plan of
B. Hepatic transaminase levels are management?
commonly in the thousands A. replace intravascular volume with
C. An erythematous maculopapular crystalloid solution and blood
rash precedes development of B. administer loop diuretics
pruritus C. initiate dopamine drip
D. It is associated with abnormally D. provide a one-time intravenous plus
elevated serum bile acid levels of crystalloid solution

85. What is the typical inheritance pattern of 88. Which of the following statements is true
mutation in the spectrin gene that results in regarding inflammatory bowel disease in
hereditary spherocytosis? pregnancy?
A. Autosomal dominant A. Pregnancy increases the likelihood
B. Mitochondrial of a flare
C. X-linked dominant B. Relapses are usually mild
D. Autosomal recessive C. All of the choices
D. None of the choices
86. Which of the following endoscopic E. Most treatments are discontinued
diagnostic procedures is contraindicated during pregnancy
during pregnancy in general?
A. Upper gastrointestinal endoscopy 89. Active disease in early pregnancy
B. percutaneous endoscopic decreases the likelihood of poor pregnancy
C. gastrostomy (PEG) tubes outcome
D. ERCP With severe lung injury and high
E. large bowel, flexible sigmoidoscopy intrapulmonary shunt fractions, which of the
F. videocapsule endoscopy for small- following is successful in decreasing the
bowel evaluation shunt by recruiting collapsed alveoli?
A. Positive End-Expiratory Pressure
87. A 30 year old G1P0 at 38 weeks age of of 5 to 15 mm Hg
gestation present to labor and delivery B. Intravenous Fluids
complaining go headache and severe C. Extracorporeal Membrane
epigastric pain with decreased fetal Oxygenation
movement. Her initial blood pressure is D. All of the choices
95. Gestational thrombocytopenia is usually
90. Is an X linked recessively transmitted evident in the third trimester and is thought
disease characterized by a marked to be predominantly due to ____.
deficiency of small component all of the choices
antihemophilic factor (factor VIII). A. none of the choices
A. Hemophilia B B. Hemolysis
B. Hemophilia C C. Hemodilution
C. Christmas disease
D. Hemophilia A 96. In pemphigoid gestatonis, there is a
reaction between maternal immunoglobulin
91. What is the most common cause of G (IgG) and which of the following?
intestinal obstruction in pregnancy and the A. Collagen V
puerperium? B. Collagen XVII
A. Intussusception C. Collagen XV
B. Carcinoma D. Collagen X
C. Adhesions
D. Volvulus 97. All EXCEPT which of the following
statements regarding autoimmune
92. What is the most reproducible findings hemolytic anemia in pregnancy are true?
in a A. Pregnancy can accelerate hemolysis
pregnant woman with appendicitis? B. The indirect Coombs test is usually
Persistent abdominal pain and tenderness positive
A. Fever C. The direct Coombs test is usually
B. Nausea and Vomiting positive
C. Anorexia D. The cause of aberrant antibody
D. Leukocytosis production originates from fetal
microchimerism
93. Sickle-cell trait (hemoglobin AS) has
been associated with an increased risk for 98. What is the initial choice of treatment for
which of the following? hidradenitis suppurativa?
A. Urinary tract infection A. Oral fluconazole
B. Placenta abruption B. Oral antimicrobials
C. Preeclampsia C. Coal tar
D. Gestational diabetes D. Oral corticosteroids

94. Which of the following is the most 99. Which of the following pregnancy-
common single defect of congenital rubella? associated complications has the greatest
Mental retardation need for intensive care
A. Cataracts A. pneumonia or asthma
B. Sensorineural deafness B. hemorrhage
C. Hepatosplenomegaly C. thyrotoxicosis
D. heart disease
E. diabetes
LONG EXAMINATION - 6
(2021 - 2022)
SURE
UNSURE

1. Ms Heidi was seen to have a 4x4cm D. justice


ovarian cyst on ultrasound. for her case, she
was advised that she may have surgical or 4. Katie, a 19 y/o student found out she was
medical management. Since the mass is still 7 weeks pregnant. Fearful of what her
not very big, she may opt for medical parents will say, she took pills to induce
management. but if she wishes so, she may abortion. In inducing abortion, what
undergo surgery. If she opted for surgical obligation was violated?
management, What principle was practiced? A. Autonomy
A. autonomy B. Beneficence
B. beneficence C. Non-Malificence
C. non-malificence D. Justice
D. justice E. All of the above

2. Ms Aila is a 35 year old nulligravid 5. Anjie, a 24 y/o G2P0 (0010) went to the
contemplating on elective freezing of her emergency room for abdominal pain. An
oocytes. if she did pursue with this, what ultrasound was requested and showed an
principle was practiced? adnexal mass on the left measuring 2.3 x
A. autonomy 2.3cm, probably an ectopic pregnancy. She
B. beneficence was offered to choose from medical
C. non-malificence treatment to laparoscopic salpingostomy
D. justice (preservation of tubes). This practices the
principle of __________.
3. Ms Heidi was seen to have a 4x4cm A. Autonomy
ovarian cyst on ultrasound. for her case, she B. Beneficence
was advised that she may have surgical or C. Non-malificence
medical management. Since the mass is still D. Justice
not very big, she may opt for medical E. All of the above
management. but if she wishes so, she may
undergo surgery. Even though she opted for 6. LB, on her 34th week AOG, was admitted
surgical management, her OB told her to try for very high BP - 220/120mm Hg. if her BP
medical management first as it is as effective remains uncontrolled, she was told that she
as surgery, and cheaper. What principle was needs to undergo emergency CS despite
practiced? having a premature baby. if LB chose not to
A. autonomy undergo the cesarean section, what principle
B. beneficence was applied?
C. non-maleficence A. autonomy
B. beneficence 10. It is the moral obligation to act on the
C. non-maleficence basis of fair adjudication between competing
D. justice claims.
A. autonomy
7. Ms Heidi was seen to have a 4x4cm B. beneficence
ovarian cyst on ultrasound. for her case, she C. non-malificence
was advised that she may have surgical or D. justice
medical management. Since the mass is still
not very big, she may opt for medical 11. Shiela is a 7 year old girl who needs her
management. but if she wishes so, she may left ovary removed. Since she is a minor, her
undergo surgery. She opted for surgical consent is not necessary even if she can
management but her OB advised against it already understand her situation
because of the risks surgery entails. What A. True
principle was practiced? B. False
A. autonomy C. Maybe
B. beneficence D. Depends
C. non-maleficence
D. justice 12. LB underwent emergency cesarean
section and was given an emergency slot at
8. Katie, a 19 y/o student found out she was the OR. Despite of the prematurity of the
7 weeks pregnant. Fearful of what her baby, her OB pushed through the procedure
parents will say, she took pills to induce to save them both. what principle was
abortion. In inducing abortion, what applied?
obligation was fulfilled? A. autonomy
A. Autonomy B. beneficence
B. Beneficence C. non-maleficence
C. Non-Malificence D. justice
D. Justice
E. All of the above 13. Katie, a 19 y/o student found out she was
7 weeks pregnant. Fearful of what her
9. Clara was admitted for labor pains. She is parents will say, she took pills to induce
on her 8th hour of labor without any progress abortion. What principle was violated?
in her cervical dilatation for two hours. Her A. Autonomy
baby's heart beat is ok. She was informed B. Beneficence
that if there will be no progress after an hour, C. Non-Malificence
she will need to undergo CS. What principle D. Justice
is applied? E. All of the above
A. autonomy
B. beneficence 14. Ysabel consulted at the ER for
C. non-maleficence occasional hypogastric pain. Afraid that she
D. justice is in labor, she consulted and asked to be
admitted. Due to the limited hospital
capacity, Lisa was sent home despite her
plea to be admitted after she was examined. not very big, she may opt for medical
what principle was applied? management. but if she wishes so, she may
A. autonomy undergo surgery. Suppose Ms Dee chose to
B. beneficence have medical management, What principle
C. non-malificence was practiced?
D. justice A. autonomy
B. beneficence
15. Anjie, a 24 y/o G2P0 (0010) went to the C. non-malificence
emergency room for abdominal pain. An D. justice
ultrasound was requested and showed an
adnexal mass on the left measuring 2.3 x 18. Anjie, a 24 y/o G2P0 (0010) went to the
2.3cm, probably an ectopic pregnancy. She emergency room for abdominal pain. An
was offered to choose from medical ultrasound was requested and showed an
treatment to laparoscopic salpingostomy adnexal mass on the left measuring 2.3 x
(preservation of tubes). The admitting 2.3cm, probably an ectopic pregnancy. She
physician is highly recommending the patient was offered to choose from medical
to undergo laparoscopic salpingostomy treatment to laparoscopic salpingostomy
under the principle of __________________. (preservation of tubes). Ms Anjie asked for
A. Autonomy possible explore laparotomy but the
B. Beneficence physician recommends against it. This
C. Non-malificence shows the principle of _________
D. Justice A. Autonomy
E. All of the above B. Beneficence
C. Non-maleficence
16. Clara was admitted for labor pains. She D. Justice
is on her 8th hour of labor without any E. All of the above
progress in her cervical dilatation for two
hours. Her baby's heart beat is ok. She was 19. Ms Heidi was seen to have a 4x4cm
informed that if there will be no progress after ovarian cyst on ultrasound. for her case, she
an hour, she will need to undergo CS. What was advised that she may have surgical or
obligation is applied? medical management. Since the mass is still
A. obligation of autonomy of the not very big, she may opt for medical
physician to the mother management. but if she wishes so, she may
B. obligation of autonomy of the undergo surgery. What principle was
physician to the mother and fetus practiced?
C. obligation of beneficence of the A. autonomy
physician to the mother and fetus B. beneficence
D. obligation of beneficence of the C. non-malificence
physician to the mother D. justice

17. Ms Heidi was seen to have a 4x4cm 20. Anjie, a 24 y/o G2P0 (0010) went to the
ovarian cyst on ultrasound. for her case, she emergency room for abdominal pain. On
was advised that she may have surgical or history, it was revealed that her first
medical management. Since the mass is still pregnancy was also an ectopic pregnancy
where her fallopian tube was removed. tocolytics which she did not take. What
Thinking of possible sterility if she undergoes principle/obligation did she violate?
another operation, she refuses to be A. Autonomy
admitted. What principle is shown here? B. Beneficence
A. Autonomy C. Non-Maleficence
B. Beneficence D. Justice
C. Non-malificence E. All of the above
D. Justice
E. All of the above 24. Jean, a 17 y/o primigravid went to an
obstetrician asking for elective termination of
21. Ms Anjie was scheduled for operative her unwanted pregnancy at 6 weeks. The
laparoscopy (minimally invasive surgery). obstetrician refused to cooperate. What
She was about to be called at the OR when principle was practiced?
her procedure was deferred. Another patient A. autonomy
was admitted and was directly admitted at B. beneficence
the OR for an emergency pelvic lap. Which C. non-maleficence
principle was applied? D. justice
A. Autonomy
B. Beneficence 25. Anjie, a 24 y/o G2P0 (0010) went to the
C. Non-Malificence emergency room for abdominal pain. On
D. Justice history, it was revealed that her first
E. All of the above pregnancy was also an ectopic pregnancy
where her fallopian tube was removed.
22. LB underwent emergency cesarean Thinking of possible sterility if she undergoes
section and was given an emergency slot at another operation, she refuses to be
the OR. Despite of the prematurity of the admitted. The admitting physician patiently
baby, her OB pushed through the procedure explained that her tubes may be saved if the
to save them both. what obligation was tubes are still intact. She was also educated
applied? on possible alternative fertility treatments if
A. obligation of autonomy of the the remaining tube would be removed also.
physician to mother This shows _____
B. obligation of autonomy of the A. Autonomy
physician to mother and fetus B. Beneficence
C. obligation of beneficence of the C. Non-malificence
physician to mother and fetus D. Justice
D. obligation of non-maleficence of the E. All of the above
physician to mother and fetus
26. JJ, an obese patient for BTL, what is the
23. Katie, a 19 y/o student found out she was best anesthesia for her in the procedure?
7 weeks pregnant. Fearful of what her A. General Anesthesia
parents will say, she took pills to induce B. Local lidocaine infiltration with IV
abortion. The attempt however failed. The sedation
parents came to know of the pregnancy and C. Local lidocaine infiltration
sent her to a physician. She was given D. Epidural anesthesia
C. Repeat CBC
27. What is the most commonly mistakenly D. Proceed BTL as scheduled
transected anatomy instead of the fallopian
tube in BTL? 32. When is the recommended timing in
A. appendix doing female sterilization?
B. intestine A. all of the choices
C. round ligament B. interval - D28 onward or during
D. ureter menstruation or anytime as long as
not pregnant -on FP methods
28. L.H., UNDERGOING A CS WITH BTL C. postpartum – D0 to D6 and post
PROCEDURE BUT DURING THE abortal
PROCESS, THE OBSTETRICIAN HAD D. at the time of other pelvic or
DIFFICULTY IN DOING THE SURGERY abdominal surgery
DUE TO MASSIVE ADHESIONS.WHICH
OF THE FOLLOWING IS MOST LIKELY 33. When is interval BTL done?
MISTAKEN AS THE FALLOPIAN TUBE? A. Day 28 onwards
A. intestine B. Day 0 to Day 6
B. round ligament
C. appendix 34. Tubal ligation is 100% effective but tubal
D. ureter occlusion is not.
A. True
29. O.P., A MULTIGRAVID, 38 YEAR OLD B. False
MOTHER JUST DELIVERED VAGINALLY 3
DAYS AGO AT HOME, WHAT IS THE 35. What is the main reason why there is a
INCISION BEST FOR HER? need for histopathologic study of the tubal
A. suprapubic segment after BTL?
B. infraumbilical A. for health insurance claims
B. to confirm transection
30. J.J., 30 YEARS OLD, G10P10( 9-0-1-9) C. it is part of the standard operating
UNDERWENT CURETTAGE FOR system
INCOMPLETE MISCARRIAGE.SHE D. to avoid litigation
SIGNED FOR BTL, WHAT IS THE BEST
BTL PROCEDURE on her? 36. Which of the following BTL method is the
A. interval most commonly used?
B. post partum A. Laparoscopy/ Hysteroscopy
B. Minilap
31. GH, G4P4(4004) who wishes to have C. All of the choices
BTL .During her recent delivery,she had D. Colpotomy/Culdoscopy
transient uterine atony.What is the best next
or immediate step? 37. Identify the procedure (F):
A. Defer BTL and reschedule after 1 Captionless Image
month of hematinics
B. Give other family planning method A. Vasectomy
like IUD B. Castration
C. BTL 43. What is/are the indication/s for
sterilization?
38. Post partum BTL: infraumbilical incision:: all of the choices
Interval BTL:____________ medical indication - heart disease, endocrine
midline suprapubic problem, genetic history of familial traits for
2 cm infraumbilical congenital anomalies
paramedian family completed: qualification by ACOG -
transverse suprapubic women 30y/o w/ 4 living children
obstetrical indication-previous uterine scar
39. Which of the following is true about
vasectomy? 44. What is the most widely used method for
A. Semen should be checked until 2 female sterilization?
consecutive sperm counts are zero A. Colpotomy
B. All of the choices B. Minilap
C. Patient have to wait 15-20 ejaculation C. Laparoscopy
before the tract becomes devoid of D. Culdoscopy
sperm.
D. Sterility is not immediate (not an 45. K.G., 40 YEAR OLD MALE
instant sterility) UNDERWENT VASECTOMY.SINCE
STERLITY IS NOT IMMEDIATE, WHAT IS
40. WHICH OF THE FOLLOWING BTL THE BEST HOME INSTRUCTION SHOULD
TECHNIQUE GIVES THE HIGHEST BE DONE?
FAILURE RATE? A. Semen should be checked until 2
A. Postpartum partial salpingectomy consecutive sperm counts are
B. Bipolar coagulation zero
C. Silicone band B. No need to have semen analysis, just
D. Spring clip take note of the number of
ejaculations
41. IN BTL WHEN CONTRACETION IS C. Semen should be checked once and
ACHIEVED? assured that it counts are zero
A. AFTER A YEAR D. Use barrier technique as double
B. AT ONCE protection
C. AFTER 6 MONTHS
D. AFTER 3 MONTHS 46. Identify type of BTL:
A. post partum
42. WHICH OF THE FOLLOWING IS A B. interval
CONTRAINDICATION FOR BTL?
A. OBESITY 47. IN TUBE REVERSAL AFTER BTL,
B. BELOW 30 YEARS OLD AND WHAT IS THE AVERAGE RETURN OF
UNCOMPLICATED FERTILITY?
C. HEART DISEASE A. AFTER 6 MONTHS
D. GRAVIDA 4 B. AT ONCE
C. AFTER 3 MONTHS
D. AFTER A YEAR
postpartum
48. WHICH OF THE FOLLOWING IS
IRREVERSIBLE BTL? 55. What is the best BTL surgical approach
A. UCHIDA in obese patients?
B. MADLENER A. infraumbilical
C. POMEROY B. suprapubic
D. SALPINGECTOMY C. Colpotomy
D. Minilap
49. BTL AND VASECTOMY ARE
CONSIDERED CASTRATION. 56. WHICH OF THE FOLLOWING BTL
A. True TECHNIQUE GIVES THE HIGHEST
B. False ECTOPIC RATE?
A. Silicone band
50. Which of the following is a side effect of B. Postpartum partial salpingectomy
BTL? C. Bipolar coagulation
A. menorrhagia D. Spring clip
B. ectopic pregnancy
C. all of the choices 57. When is Post abortal BTL done?
D. chronic pelvic pain A. Day 0 to Day 6
E. decreased libido B. Day 28 onwards

51. AT WHAT FALLOPIAN TUBE 58. AS A RESIDENT PLANS FOR BTL,


SEGMENT IS BTL MOST COMMONLY WHAT TECHNIQUE IS CONSIDERED THE
DONE? LEAST LIKELY TO HAVE ECTOPIC
A. ISTHMUS PREGNANCY?
B. AMPULLA A. Bipolar coagulation
C. FIMBRIA B. Spring clip
D. INTERSTITIAL C. Postpartum partial salpingectomy
D. Silicone band
52. VASECTOMY IS MORE EFFECTIVE AS
CONTRACEPTION THAN BTL. 59. BTL and Vasectomy interfere with sex
A. True hormone production .
B. False A. True
B. False
53. PREGNANCY TEST IS A
PREREQUISITE TO BTL. 60. Non surgical BTL approach
1 point A. culdotomy
A. True B. culdoscopy
B. False C. hysteroscopy
D. colpotomy
54. In what BTL procedure is the instrument E. laparoscopy
below used?
Captionless Image 61. Who among the following is amenable for
interval interval BTL?
A. Mary, weighing 110kg A. take elevated dose of COC within
B. Karen, uncomplicated post abortion 72 hours followed by a second
C. Joey, uncomplicated post partum dose of 12 hours later
D. Fitzy, with history of
endometriosis and now with non 67. This determines the way a pregnant
movable uterus woman reacts to the dynamic changes in
E. Rosita, with hemoglobin of 50 grams pregnancy:
A. Physical
62. When is Post partum BTL done? B. Spirit
A. Day 28 onwards C. Psyche
B. Day 0 to Day 6 D. Soma

63. H.J., A G4P4(4001) WHO JUST GAVE 68. Period of Acceptance is embodied during
BIRTH TO HER 4TH TURNER'S this stage:
SYNDROME BABY.SHE FINALLY A. First Trimester
DECIDED TO HAVE STERILIZATION, B. Second Trimester
WHAT IS THE CRITERIA MET BY THE C. Labor
PATIENT FOR BTL? D. Third Trimester
A. Psyche
No options. But maybe it cause be for the 69. Post-partum Blues is observed usually
reason of Caution or Precaution how many days after delivery?
A. 4 days
64. The emotional needed when a pregnant B. 7 days
mother starts to become childish, is: C. 2 day
A. Reassurance D. 1 day
B. All of the choices E. Option 6
C. Understanding
D. Support 70. The time where there is love or
resentment towards child for the feeling of
65. In a 28-day pill, the last 7 white pills being controlled:
usually contain which of the following? A. Parturition
A. Cyproterone acetate B. Third Trimester
B. Lactose C. First Trimester
C. Ferrous Fumarate D. Second Trimester
D. Ferrous Sulfate
71. Post-partum blues can be accounted to
66. Which of the following is/ are the proper be secondary to which of the following?
use of Emergency Contraception pills? A. All of the choices
take elevated dose of COC within 96 hours B. Progesterone withdrawal
followed by a second dose of 12 hours later C. Effect of fatigue and physical stress
take elevated dose of COC within 72 hours caused by labor
followed by a second dose of 24 hours later D. Strange hospital surroundings and
take elevated dose of COC within 96 hours unfamiliar people
followed by a second dose of 24 hours later
72. Emotional Equanimity is seen during: 77. The aspect which harbors consciousness
A. Third Trimester and labor and free will, is:
B. First and Second Trimester A. All of the choices
C. Second Trimester and Postpartum B. Soma
Period C. Spirit
D. Second and Third Trimester D. Psyche

73. Which of the following is a proper use of 78. The MEC Wheel category where a
barrier method? chosen family planning is generally used, is
A. Diaphragm should be removed for the:
at least 6 hours after intercourse A. Category 3
so as to avoid Toxic Shock B. Category 4
Syndrome C. Category 1
B. DURATION of maximal spermicidal D. Category 2
effectiveness is 1 hour
C. Spermicide should be placed in the 79. Post-Partum Depression can be
upper vagina for protection and be prevented by:
repeated before each coital episode. A. Secured Environment
D. All of the above B. Stress
C. adequate food intake
74. The Emotional Valor is seen at what part D. Unplanned Pregnancy
of the pregnancy?
A. Second Trimester 80. Which of the following is NOT a Natural
B. Labor Family Planning Method?
C. Third Trimester A. Lactation Amenorrhea Method
D. Post Partum Period (LAM)
B. Rhythm Method
75. The peak of emotional tension curve is C. Billing’s Method
seen during: D. Coitus Interruptus
A. Third Trimester
B. Labor and delivery 81. The Integrated Medical Psychology
C. Second Trimester operates in this manner:
D. Post partum period A. Bipartite Aspect
E. First Trimester B. Single Aspect
C. Tripartite Aspect
76. Progesterone Implants are administered D. Quartite Aspect
via what route?
A. Intramuscular 82. Family Planning of choice given to
B. Intradermal lactating women , except:
C. Subdermal A. Progestin Only Pills
D. Dermal B. Implants
E. Intra-cutaneous C. IUD
D. Combined Depo Injection
83. Post Partum IUD insertion should not be D. First Trimester
done in which of the following?
A. Ruptured Bag of waters more than 89. Which of the following is a complication
18 hours, without fever of Hyperemesis Gravidarum?
B. Inserted after 48 hours after delivery A. Hepatocellular Jaundice
and breastfeeding B. All are correct
C. All are correct C. Intrauterine Fetal Growth
D. Post partum hemorrhage with D. Encephalopathy
AMSTL done
90. Which of the following is a NON-
84. Nausea and vomiting spontaneously HORMONAL Family Planning Method?
start to resolve at what age of gestation? A. Dermal Patch
A. 6-8 weeks B. Copper T380 IUD
B. 8-10 weeks C. Mirena IUD
C. 12-14 weeks D. Implants
D. 24 weeks E. Oral Contraceptive Pills

85. The Emotional Valor is seen at what part 91. Which of the following is NOT a “client
of the pregnancy? dependent” method?
A. Post Partum Period A. Pills
B. Second Trimester B. All of the above is correct
C. Labor C. LAM
D. Third Trimester D. IUD
E. Condom
86. Which is a characteristic disadvantage of
a MULTIPHASIC Oral Contraceptive Pill? 92. This type of emotion is involved during
A. Difficulty in “Doubling Up” Quickening:
B. Decrease in breakthrough bleeding A. Fantasy
C. Lesser Confusion B. Acceptance
D. Has varying amounts of Estrogen C. Valor
and Progestin D. Fear

87. What is/are the absolute 93. A common complication of


contraindication/s of IUD? Methamphetamine Drug abuse?
A. Pregnancy A. Abruptio
B. Abnormal, Irregular, Uterine bleeding B. Placenta Accreta
C. All of the choices C. Hyperemesis Gravidarum
D. Active, Recent, Recurrent Pelvic D. Fetal macrosomia
Infection
94. Which of the following is NOT a beneficial
88. A time of most physical discomfort is: effect of COC?
A. Post partum period A. Prevents Cervical Cancer
B. Second Trimester B. Improves Bone mineral density
C. Third Trimester C. Decrease epithelial ovarian cancer
D. Lowers PID Incidence 101. Oral contraceptive Pills are ideally
started during which of the following?
95. This type of emotion is involved during A. 3 months after childbirth who are
Quickening: breastfeeding
A. Valor B. 1 week after an abortion
B. Acceptance C. First Day after the menstruation stops
C. Fear D. All of the above is correct
D. Fantasy E. 6 months after delivery if not
breastfeeding
96. Pica is normally seen in which of the
following?
A. All are correct
B. Mentally Challenged Individuals
C. Young Children
D. Pregnancy

97. Which is NOT a complication of


Substance Abuse?
A. Small for Gestational Age
B. Post Term Pregnancy
C. Fetal Alcohol Syndrome
D. Stillbirth

98. The period with sensory overloading, is


during:
A. Third Trimester
B. Post Partum Period
C. Second Trimester
D. Labor

99. This condition is similar to a Male


Pseudocyesis:
A. Hyperemesis Gravidarum
B. Factitious Pregnancy
C. Pica
D. Couvade

100. What is/are contraceptive choice/s for


women past 35?
A. DMPA injectable
B. Progestin implants
C. IUD
D. All of the above
MIDTERM
(2021 - 2022)
SURE
UNSURE

1. This antepartum surveillance method improves outcome of twin pregnancy:


A. Ultrasound evaluation every 2 weeks
B. Doppler velocimetry for umbilical artery
C. Biophysical scoring
D. NST 1x a week

2. 23 year old G1P0 (0010) s/p suction curettage for hydatidiform mole came in due to profuse
vaginal bleeding 3 months post molar evacuation. Pregnancy test positive. Serum beta Hcg done
revealed 102,000 miUL/ml/. Transvaginal ultrasound done showed an endomyometrial mass
measuring 5.4x5 x4 cm with full thickness myometrial invasion. What is your impression?
A. Incomplete evacuation of molar pregnancy
B. Another Hydatidiform Mole
C. Endometrial Cancer
D. Gestational Trophoblastic Neoplasia

3. Most variable biometric measurement


A. Placental grade
B. Abdominal circumference
C. Femoral length
D. Biparietal diameter

4. The biometric parameter influenced by shape of the uterus


A. Femoral length
B. Abdominal circumference
C. Head circumference
D. Placental grade

5. Which of the following is/are predictor/s of successful induction of labor ?


A. Bishop score > 6 and prior vaginal delivery
B. Active genital herpes
C. Abnormal fetal lie or presentation
D. Prior classical or inverted T uterine incision
E. Placenta or vasa previa or cord presentation

6. Which of the following is associated to reduce the incidence of preeclampsia?


A. Aspirin
B. Vitamin E
C. Calcium supplements
D. Ascorbic acid

7. Which condition triggers the FAST PATHWAY of coagulation disorders?


A. Septic Abortion
B. Abruptio placenta
C. Eclampsia
D. Chorioamnionitis
E. HELLP Syndrome

8. Human fetal growth is characterized by sequential patterns and divided into three phases. This
phase is a phase of cellular hyperplasia and hypertrophy
A. 16 - 28 weeks age of gestation
B. 16 - 32 weeks age of gestation
C. 0-16 weeks age of gestation
D. more than 32 weeks age of gestation

9. Which of the following is TRUE regarding management of DIC in Pregnancy?


A. Laboratory tests are specific and routinely requested
B. There is a case-to-case management scenario
C. Clinical trials can be easily conducted for clinical evidences
D. Option 6
E. Antifibrinolytics can be used in the management all the time

10. J.J G2P1 (1011) GTN Stage I:10 . What chemotherapeutic drug will you give?
A. Actinomycin
B. Methotrexate
C. EMACO
D. Etoposide-Atinomycin

11. Which maternal infection issues the risk of having hypertension in pregnancy?
A. PTB
B. Dental carries
C. Community acquire pneumonia
D. Dengue Fever

12. Mrs JA came in at 35 weeks for a fetus small for gestational age. which of the following
necessitates immediate delivery
A. Amniotic fluid of 12cm
B. All of the choices
C. Reassuring fetal heart rate pattern
D. Reversed end diastolic flow in umbilical studies

13. Early onset preeclampsia is evidently seen in which of the following condition?
A. Diabetes Mellitus
B. Molar Pregnancy
C. Multiple Gestations
D. Hyperemesis gravidarum

14. Which of the following is a risk factor for placenta previa?


A. Obesity
B. Age < 20 years old
C. Gestational Diabetes mellitus
D. previous cesarean section

15. Patient JK is a G4P2 full term, frank breech in labor.Which of the following forceps is ideal to
deliver the aftercoming head?
A. Piper forceps
B. Tucker-McLane forceps
C. Kielland forceps
D. Simpson forceps

16. Which of the following is/are amenable to planned vaginal breech birth?
A. Hyperextended fetal head
B. Cord presentation
C. Frank breech presentation
D. Fetal growth restriction
E. All of the choices

17. This is the most common vascular anastomosis in monochorionic twin placenta:
A. Deep Vein to vein anastomosis
B. Deep artery to vin anastomosis
C. Superficial Artery to venous anastomosis
D. Superficial Artery to artery anastomosis

18. To whom in the following is induction of labor most risky?


A. PROM
B. Pregnancies ≥ 41 weeks
C. Previous CS
D. Intrauterine fetal death
E. Group B Streptococcus positive

19. Fetal growth phase that involves cellular hyperplasia and hypertrophy starts at ____ weeks
A. 17
B. 10
C. 32
D. 14
E. 12

20. Which of the following factors does NOT affect the resting blood pressure reading?
A. Activity level
B. Gravidity
C. Age and weight
D. Race

21. True of tachysystole EXCEPT:


A. Inadequate resting tone
B. “hyperstimulation” OR “hypercontractility
C. 5 contractions in 10 minutes, averaged over 30 minutes
D. Prolonged contraction: lasting > 90 seconds

22. An adverse outcome unique to fetal overgrowth


A. Perineal laceration
B. All are unique to fetal overgrowth
C. Postpartum hemorrhage
D. Shoulder dystocia

23. Which of the following statements about molar pregnancies is TRUE?


A. Vaginal bleeding is a common symptom of hydatidiform mole
B. The most common chromosomal make up of a partial or incomplete mole is 46 XX or paternal
origin
C. Partial H mole has higher risk of developing into choriocarcinoma than complete mole
D. Older maternal age is not a risk factor for hydatidiform mole

24. TRUE of Doppler velocimetry


A. Abnormal umbilical artery Doppler velocimetry findings have been uniquely linked with
fetal-growth restriction
B. Late changes are characterized by normal flow in the ductus venosus and abnormal in fetal aortic
and pulmonary out flow tracts and by reversal of umbilical artery flow.
C. Early changes in placenta-based growth restriction cannot be detected in peripheral vessels such
as the umbilical and middle cerebral arteries.
D. All of the choices

25. Which of the following uterotonic is the safest and has the least side effects?
A. Oxytocin
B. Carbetocin
C. Carboprost
D. Methrylergometrine maleate
E. Misoprostol

26. When is elective induction of labor recommended?


A. >/=40 weeks AOG
B. >/=41 weeks AOG
C. >/=38 weeks AOG
D. >/=42 weeks AOG

27. Which of the following is a prerequisite of outlet assisted vaginal delivery?


A. Scalp is visible without separating the labia
B. Fetal skull has reached the pelvic floor
C. All of the choices
D. The sagittal suture is in anterior-posterior diameter

28. TRUE OF PREVENTION of fetal growth restriction except:


A. Smoking cessation not needed
B. Optimize maternal condition
C. Begins as soon as diagnosed
D. Treatment of hypertension
E. none of the choices
29. Mrs JA came in at 35 weeks for a fetus small for gestational age. Which of the following will allow
observation?
A. growth of 200g /week
B. Increased heart rate
C. None of the choices
D. Increased resistance of maternal arteries

30. Mrs A has a fundic height of 35cm, with ultrasound results showing fetal weight estimate of 4000g. if
she underwent vaginal delivery, possible complications are?
A. hysterectomy
B. Clavicular fracture of baby
C. none of the choices
D. hemorrhage
E. all of the choices

31. What is the accepted Radiologic examination to delineate High Order Multiple Gestations?
A. MRI
B. Abdominal Xray
C. Ultrasound -trans
D. CT Scan

32. Why do we use population reference in determining normal birth weight?


1. none of the choices
2. all of the choices
3. Incorporates resulting outcomes of normal and abnormal pregnancies
4. incorporates pregnancies of varying risks and resulting normal outcomes - williams page
873
5. Incorporates normal pregnancies with normal outcomes

33. Mrs A is suspected to have fetal macrsomia. You will advise...


A. Labor induction at 38 weeks age of gestation to prevent stillbirth
B. All choices are correct
C. Current evidence does not support a policy for early labor induction before 39 weeks’
gestation or delivery for suspected macrosomia.- trans
D. Labor induction will reduce the risk of shoulder dystocia and cesarean delivery.

34. What is the underlying etiology of proteinuria seen in preeclampsia?


A. Increased vascular resistance
B. Increased renal artery resistance
C. Increased Glomerular Filtration rate
D. Increased capillary permeability

35. Patient MA a 29-year-old G5P4(4004), came to the ER with vaginal bleeding 5 days after going to a
traditional birth attendant or “manghihilot”. At present, she has hypotensive blood pressure, tachycardic
and febrile at 38.6 C . She also had foul smelling discharges. She eventually had symptoms of DIC.
Which of the following is the pathway activated during this condition?
A. Abnormal platelet aggregation pathway
B. Extrinsic Pathway
C. Intrinsic Pathway
D. Common Pathway

36. Cleavage during this day after fertilization would lead to formation of cojoined twins:
A. Day 14
B. Day 3
C. Day 7
D. Day 12

37. Mrs A is now admitted for labor. she has a fundic height of 35cm, with an ultrasound result confirming
singleton pregnancy. Latest ultrasound showed fetal weight estimate of 4000g, with amniotic fluid of 3cm.
Which is of the following will make u lean towards cesarean section?
1 point
Amniotic fluid of 3cm
Fundic height of 35cm
Any of the choices
Sonographic estimate of 4000g

38. During abnormal platelet activation, which of the following is decreased?


1 point
Prothrombin
Thrombin
Prostacyclin
Fibrinogen

39. Mrs. Bakersman, 35 year old, G2P1 (1001) gravidocardiac. She underwent normal labor but was
delivered by assisted vaginal delivery. Completion of third stage followed quickly and the fundus was
noted to firm however, brisk vaginal bleeding as still noted. What is the most likely cause of the bleeding?
1 point
Uterine atony
Uterine rupture
Retained placenta
Genital tract laceration

40. Obstetrical dilemma postulates that ________


1 point
The ability to walk upright needs a narrow pelvis williams page 872
Our ability to restrict our growth is pathological
A narrow pelvis compliments a large head
ALL are TRUE

41. NOT an etiology of fetal overgrowth


1 point
Excessive transfer of lipids
Excessive glycemia
Overexpressed endothelial lipase
Fetal hyperinsulinemia
Reduced modulation of rapamycin complex - trans
42. G1P0 40 wks AOG in active labor for 4 hours, with adequate contractions. Cervix 6 cms dilated,70%
effaced, cephalic, station -3; FH = 40 cms; clinical pelvimetry is adequate. 2 hrs later, cervix was 7-8 cms,
station 0; and was fully dilated after an hour but remained at station 0. The dystocia is most likely caused
by:
1 point
Powers
Soft tissue abnormalities
Passenger
Passage

43. Parameter that is uniquely linked to fetal growth restriction


1 point
Increased uterine resistance indices
All are unique to growth restriction
Decreased flow in the cerebral arteries
Abnormal umbilical Doppler velocimetry - williams page881

44. Which of the following is a WHO recommendation to prevent post partum hemorrhage?
1 point
Delayed cord clamping for 2-3 minutes or when cord pulsation stops - who website
Non separation of the mother and child
All of the choices
Skin to skin contact of the mother and child
Oxytocin 10 units IM be given 1 minute after delivery

45. Which of the following will NOT result to fetal restriction


1 point
Maternal hypertension
Multiple Pregnancy
Caffeine intake
Alcohol intake
None of the choices

What is prolonged latent phase in nullipara?


1 point
>2hr
>1hr
>14 hrs
>3hr
>20 hrs

What is the first line uterotonic?


1 point
Carbetocin
Misoprostol
Oxytocin
Carboprost
Methrylergometrine maleate

Intravascular clot disintegration is accomplished by which of the following in maintaining the hemostatic
balance?
1 point
Unobstructed blood flow
Action of prostacyclin
Intact Reticuloendothelial system
Presence of humoral factors

Ms Anita came in for prenatal check up at 32 weeks AOG. During this time, the estimated fetal weight is
1900g. She returned at 35 weeks AOG. the expected fetal weight should be
1 point
2100g - 2200g
2400g - 2500g
2200g - 2300g
2300g - 2400g

In growth restricted fetus, when is the best time to give antenatal corticosteroids?
1 point
Before 34 weeks age of gestation
Before 32 weeks age of gestation
Before 36 weeks age of gestation
Before 30 weeks age of gestation

What is the first surgical step in case of retractable uterine atony?


1 point
total hysterectomy
Ligation of uterine and ovarian arteries
uterine artery embolization
Ligation of hypogastric arteries

Prophylactic labor induction in fetal overgrowth is advised….


1 point
…if the fetus has an estimated fetal weight of 4500g
…in non diabetic mother as soon as baby reaches term
….to reduce risk of shoulder dystocia
..for suspected macrosomia at 38 weeks age of gestation
Which hormone increases the incidence of twinning in certain ethnic groups?
1 point
LH
FSH
Progesterone
Estrogen
May be both a cause and consequence of hypoperfusion at the placental site.
1 point
Implantation site disorders
Fetal growth restriction
Placental abnormalities
None of the choices
Pre eclampsia Syndrome

Most reproducible biometric parameter/ measurement


1 point
Femoral length
Biparietal diameter
Abdominal circumference
Head circumference
.
All of the following statements about theca lutein cyst are Correct, EXCEPT:
1 point
It is due to hyperstimulation by high levels of circulating βhCG, in up to 50% of cases
It is surgically removed once present in h. mole
Option 6
It will regress after 8-12 weeks post suction curettage

There is 30% of occurrence of theca lutein cyst in complete h.mole


Px DD, 35 year old gave birth 5 days ago came back due to fever with associated foul smelling lochia and
hypogastric tenderness. What is the diagnosis?
1 point
subinvolution of the uterus
postpartum endometritis
pyelonephritis
puerperal sepsis
cystitis

If Alita was found to have a baby with small for gestational age at 34 weeks, when will you repeat
sonographic assessment of fetal weight?
1 point
After a week
After four weeks
After three weeks
After two weeks
Which of the following is best for fetal growth restriction prevention?
1 point
In patients with previous babies with fetal growth restriction, Doppler studies are recommended
Low dose aspirin prevents recurrence of fetal growth restriction
Accurate dating ideally during early pregnancy
Fetal growth restriction begins as soon as detected
This is the leading cause of DIC in pregnancy:
1 point
Acute obstetrical hemorrhage
Amniotic fluid embolism
Hypertensive complications
Infection

61. What is third stage of labor?


Begins at 4 cms dilated to full cervical dilatation
Begins from placental delivery to 1 hour post partum
Begins immediately after fetal birth and ends with placental delivery
Begins from fetal delivery to first 4 hours post partum
Begins from full cervical dilatation to fetal expulsion

62. It is the process by which the uterus returns to its normal size, tone and position.
Uterine consistency
Uterine atony
Uterine contraction
Uterine involutionn
Uterine dehiscence

63. Patient AC, 34 year old delivered vaginally to her 5th baby and on the 3rd post partum hour, she
complained of excruciating perineal pain.On evaluation, there was about 6x8 cm vulvar hematoma.What
is the best treatment for her?
Hot sits bath
Evacuation of hematoma and repair
Wash with soap and water only
Perilight
Cold compress

64. Mrs TE is at her 36th week of gestation. On her check up, her baby was noted to be less than the 5th
percentile of the expected weight. BPP, NST and Doppler studies were normal. What is the next
appropriate step?
Repeat sonography after 3-4 weeks
Repeat NST and BPP after 3 days
Determine Amniotic fluid volume after 3 days
Give dexamethasone for fetal lung maturity for possible delivery

65. The probably of having another previously restricted growth fetus (in a mother who had a baby that
was growth restricted)
Up to 5%
Up to 15%
Up to 20%
Up to 10%

66. Which of the following is NOT usually a complication of Multifetal Gestation?


Post term delivery
Fetal malpresentation
Postpartum hemorrhage
Congenital anomalies
Preeclampsia

67. What is the average time for placental delivery?


10 minutes
5 minutes
30 minutes
1 minute
15 minutes

68. What is the preferred treatment for prolonged latent phase in nullipara?
CS
Rest and if exhausted CS delivery
Expectant and support
Bed rest
Oxytocin drip

69. Mrs RB is in her 33rd week of gestation and was found to have a very small fetus for the gestation.
Which of the following is an absolute indication for delivery?
Amniotic fluid volume of 8cm
Reverse end diastolic flow
Fetal heart rate of 165 - 170
Maternal BP 160/100

70. TRUE of role of ultrasound in assessing fetal growth


First-trimester sonography is good in predicting SGA newborns
All of the choice
first-trimester examination closely estimates age of gestation
first-trimester examination is needed to establish gestational age and identify anomalies

71. Cesarean section may be advised for non-diabetic mothers with fetus weighing
1 point
5500g
5000g
4500g

72. Depends on the mother


It is to soften, efface, or dilate the cervix:
Induction of Labour
Cervical Ripening
Augmentation of labor
Elective Induction

73. Mrs A is now admitted for labor. she has a fundic height of 35cm, with an ultrasound result confirming
singleton pregnancy. Latest ultrasound showed fetal weight estimate of 4000g, with amniotic fluid of 3cm.
Which is of the following will you Least consider to make you lean towards cesarean section?
Amniotic fluid of 3cm
Fundic height of 35cm
Any of the choices
Sonographic estimate of 4000g

74. Fetal growth restriction can lead to EXCEPT


Neonatal asphyxia
Meconuim aspiration
Neonatal hypotermia
Stillbirth
Neonatal hyperglycemia

75. Red Blood Cell destruction can be evident by looking at which diagnostic parameter, EXCEPT ?
Peripheral blood smear
Unconjugated serum bilirubin
Lactate dehydrogenase
Creatinine

76. To prevent uterine inversion, which of the following should be done?


controlled cord traction during placental delivery
Administer oxytocin 1 minute after the placental delivery
placental expression should be forced before placental separation
umbilical cord traction must be pulled from the uterus
All of the choices

77. To whom redosing of prophylactic antibiotics is not necessary?


Px VN, who has a BMI of 35
Px BL, who had a 1800cc blood loss
All of the choices
Px TL, who had CS III for 1.5 hours
Px RG, who had CS with enterolysis

78. Human fetal growth is characterized by sequential patterns and divided into three phases. This phase
is a phase of hyperplasia
16 - 28 weeks age of gestation
0-16 weeks age of gestation
16 - 32 weeks age of gestation
more than 32 weeks age of gestation

79. Which finding is least characteristic of complete hydatidiform mole?


hypothyroidism
vaginal bleeding
hypertension
large for dates uterine size

80. 32 year old G3P2 (2002) previous history of hydatidiform mole came in at the emergency room due to
palpable fluctuant mass at the distal third of the vagina. She claimed she had amenorrhea x 3 months. PT
positive. Beta hCG requested revealed >1 Million. Work ups done. Transvaginal ultrasound showed
bilateral ovarian mass probably theca lutein cyst. Chest Xray and CT scan done revealed no metastases
seen. What is the FIGO anatomic staging?
Stage II
Stage IV
Stage III
Stage I

81 p 57 kip 2 is an immunostaining that will confirm the diagnosis of partial hydatidiform mole and
hydropic placenta from complete mole. That is…
1 point
Maternally derived but paternally expressed
Negative result in partial hydatidiform mole and hydropic placenta
Paternally derived but maternally expressed
Positive result in complete hydatidiform mole

82 This incorporates pregnancies of varying risks, along with the expected outcomes - both normal and
abnormal
1 point
Mean population birthweight
Obstetrical estimate birthweight
Standard reference birthweight
Population reference birthweight

83 Which of the following is NOT TRUE regarding the Intrinsic Pathway of activating the coagulation
system?
1 point
It is a faster process
There is damage to vessel endothelium
Initially activates the Hageman factor
Endotoxins are present

84 Which of the following is NOT TRUE?


1 point
Fetal growth rate is estimated by sonographic measurement
Low birthweight always correlates with fetal growth restriction
All choices are true
A large percentage of SGA infants have normal outcomes and are thought to be appropriately
grown when maternal ethnic group, parity, weight, and height are considered

85 How do we manage the third stage of labor?


1 point
By waiting for placental separation signs
Physiological management
Active management of third-stage labor (AMSTL)
Expectant management
By the aid of nipple stimulation or gravity[ALL OPTIONS ARE CRT

86 Getting infected with this DOES NOT cause fetal growth restriction
1 point
Leptospirosis
Syphilis
Cytomegalovirus
Rubella

87 Which of the following leads to fetal overgrowth?


1 point
Reduced mTORC activity
Reduced endothelial lipase
Higher fatty acid binding proteins
Diminished maternal glucose

88 What is post partum haemorrhage?


1 point
Any time that there is no urine output
Blood loss of 500 ml or more within 24 hours after birth in both NSD and CS
Blood loss of 1000 ml or more within 24 hours after birth
Any time the postpartum hematocrit lower than one obtained on admission for delivery
Any time that there is decreased BP

89 Commonly diagnosed as missed abortion:


1 point
46 XX or 46 XY
Partial Hydatidiform Mole
negative p 57 kip 2
diandric diploidy

90 Which of the following is the last resort or therapy to be done in uterine atony?
1 point
Tamponade with Sengksten Blakemore esophageal catheter
Emergency Hysterectomy
B Lynch or Cho sutures (compress sutures)
Emergency laparotomy with pelvic vessel (Internal iliac or uterine artery) ligation
Emergency embolization

91 Mrs. Blippi 39 y. o. G10P7(7127) came in due to severe abdominal pain associated with pallor. Patient
claimed that 4 months (July 2021)) PTA, she underwent suction curettage. Ultrasound done revealed
endomyometrial mass measuring 7 x 8 x 6 cm with full thickness myometrial invasion with note of break
at the serosal layer. Serum beta hCG was 250,000.00. What could be the type of trophoblastic tumor?
1 point
Choriocarcinoma
Invasive Mole
Epitheliod type Trophoblastic Tumor
Placental Site Trophoblastic tumor

92 TRUE of amniotic fluid assessment


1 point
There is no association between amniotic fluid volume and fetal growth
Amniotic fluid is a good indicator of fetal growth
Decreased amniotic fluid volume at 24 - 34 weeks age of gestation was significantly associated
with malformation
All are TRUE

93 PROM is a frequent clinical finding in women with ineffective labor due to:
1 point
Fetal congenital anomalies
Urinary tract infection
Cephalopelvic disproportion
Oligohydramnios

94 Px U.I. G1P1 who had vaginal delivery and had freshly inverted uterus with attached placenta, what
should be done?
1 point
Any of the choices
Delay in treatment
Repostion the uterus with placenta
Remove the placenta then reposition the uterus
Start oxytocin

95 A 35-year old female consulted and claims to be about seven weeks pregnant. The ultrasound reveals
an enlarged uterus with numerous anechoic spaces (granular appearance) and no identifiable gestational
sac. What symptom will this patient most likely have presented within this setting?
1 point
increase appetite
Abdominal distention
urinary frequency
Emesis

96 What is puerperal pyrexia?


1 point
Fever including the24 hours and within first 10 days of following delivery
Fever excluding first 24 hours and within first 10 days of following delivery
A rise in temperature of 100.4°F (38°C) or more on two separate occasions at 24 hours apart
A rise in temperature of 100.4°F (38°C) or more on the first 10 days following delivery
Fever of any extent during puerperium

97 The best way to diagnose fetal growth restriction is through


1 point
maternal weight gain
Amniotic fluid measurement
uterine fundal height
serial sonographic fetal weight measurement
First trimester ultrasound

98 Importance of first trimester sonography in fetal growth


1 point
All are important
Closely estimates after of gestation
Can predict SGA infants accurately
Can identify anomalies significantly
99 Which of the following is/are criteria for having Gestational Hypertension?
1 point
Presence of thrombocytopenia
Elevated BP of at least occasions 12 hours apart
Blood pressure >140/90 after 20 weeks AOG
Proteinuria of >300mg in 24 hours

100 Who among the following will not likely have SSI?
1 point
Px LL with blood loss of 2 liters
Px CD with anemia and malnutrition
Px LM with subcutaneous thickness of > 3 cms
Px CV with BMI >30
Px OL with controlled GDM
FINALS
(2021 - 2022)
SURE
UNSURE

1. In a 28-day pill, the last 7 white pills usually contain which of the following?
A. Cyproterone acetate
B. Ferrous Sulfate
C. Lactose
D. Ferrous Fumarate

2. What is the most common nonlethal trisomy?


A. Down’s syndrome
B. Edward’s syndrome
C. Patau’s syndrome
D. Turner’s syndrome

3. Alexa, a 35 year old G11P10 (10 0 0 10) at 30 weeks aog, presents complaining of a lump
on her tongue. it is nonpainful but does bleed easily. A photograph is provided. What is
the most likely diagnosis?
A. Captionless Image
B. Condyloma
C. Syphilis
D. Neurofibromatosis
E. pyogenic granuloma

4. It explains the why some diseases appear to follow more than one type of inheritance:
A. Allelic heterogeneity
B. Genetic heterogeneity
C. Phenotypic heterogeneity
D. Locus heterogeneity

5. Oral contraceptive Pills are ideally started during which of the following?
A. 1 week after an abortion
B. 3 months after childbirth who are breastfeeding
C. All of the choices
D. First Day after the menstruation stops
E. 6 months after delivery if not breastfeeding

6. A 33 year old woman at 10 weeks gestation went to your clinic for her 1st prenatal care.
Routine laboratory tests were drawn, and result showed positive for Hepatitis B surface
antigen. Liver function tests are normal and her Hepatitis B core and surface antibody
tests are negative. Which of the following is the best way to prevent neonatal infection?
A. Provide Hepatitis B vaccine to neonate
B. Perform Cesarean delivery at term
C. Provide immune globulin to the mother
D. Provide Hepatitis B vaccine to the mother
E. Provide immunoglobulin and the Hepatitis B vaccine to the neonate

7. Characterized by a low serum TSH level with normal thyroxine hormone levels
A. Subclinical hyperthyroidism
B. Euthyroid autoimmune thyroid disease
C. Isolated maternal Hypothyroxemia
D. Subclinical hypothyroidism
E. Hyperthyroidism

8. It is a process where persistent fetal cells may engraft in the mother causing some
maternal autoimmune diseases.
A. Microchimerism
B. Imprinting
C. Mosaicism
D. Chimerism

9. Which is TRUE about chest radiography during pregnancy?


A. Lead apron shield is optional
B. Fetal radiation exposure is substantial
C. The heart silhouette is normally larger
D. Gross cardiomegaly is difficult to diagnose

10. Red Blood Cell destruction can be evident by looking at which diagnostic
parameter,EXCEPT:
A. Lactate dehydrogenaseo
B. Peripheral blood smear
C. Unconjugated serum bilirubin
D. Creatinine

11. What is the most common aneuploidy?


A. Trisomy 1
B. Trisomy 18
C. Trisomy 13
D. Trisomy 21
12. Which is the specific marker for sepsis?
A. Procalcitonin
B. Pro BNP
C. WBC
D. C reactive Protein

13. It is the moral obligation to act on the basis of fair adjudication between competing claims.
A. autonomy
B. beneficence
C. non-malificence
D. justice

14. he peak of emotional tension curve is seen during:


A. Second Trimester
B. Third Trimester
C. Post-partum period
D. Labor and delivery
E. First Trimester

15. A 22 year old pregnant woman was diagnosed with Toxoplasmosis. Which of the following
risk factors is most likely to have contributed to her diagnosis?
A. Owning a dog
B. Filipino nationality
C. Eating raw fish
D. Eating raw meat
E. Having viral infection in early pregnancy

16. Which of the following is not true about Transfusion Related Lung Injury (TRALI)?
A. Responds to diuretics treatment
B. Occurs within 1 -6 hours of transfusion
C. O2 saturation <90% on room air
D. Pulmonary edema on Chest X ray

17. Most of the Genetic disease is commonly recognized among:


A. Day 3
B. Age 18
C. Newborn
D. Age 5

18. Ancillary test that gives accurate diagnosis of most heart diseases during pregnancy.
A. Chest Radiograph
B. Echocardiography
C. Arterial blood gas
D. Electrocardiogram

19. Which is used for rapid identification of a specific chromosome abnormality and for verification
of suspected microdeletion or duplication syndromes?
A. chromosomal microarray analysis (CMA)
B. fluorescence in situ hybridization (FISH)
C. cytogenetic analysis (karyotyping)
D. Whole genome sequencing (WGS) /Whole exome sequencing (WES)

20. Which of the following is seen as septated cystic hygromas and or hydrops fetalis?
A. Down’s syndrome
B. Turner’s syndrome
C. Edward’s syndrome
D. Patau’s syndrome

21. Which of the following statements is true of pulmonary artery catheter monitoring in acutely ill
gravida?
A. It has been shown to improve survival
B. Its use in critically ill obstetrical patients is of limited value
C. It aids in the management of patients with low injury severity scores
D. It is essential for the care of patients with severe pre eclampsia

22. Which of the following is NOT a “client dependent” method?


A. IUD
B. All of the choices
C. LAM
D. Pills
E. Condom

23. In cases of hypovolemic shock, oliguria/anuria develops at a volume deficit of


A. 20%
B. 30%
C. 15%
D. 40%

24. In which condition would one should include antimicrobials in the treatment of GERD?
A. PUD
B. H. pylori positive
C. Helminths positive
D. Hyperemesis gravidarum

25. Regarding the use of high-potency topical corticosteroid, which of the ff statements is true?
A. they should only be used for 2 to 4 days
B. they are best reserved for refractory disorders
C. they should be used over extensive areas for best efficacy
D. they are the preferred agent for initial treatment of dermatological disorders

26. Px N.K., 26 weeks AOG was brought to ER due to fever, vomiting and hematuria. Pertinent
PE revealed costovertebral tenderness. What is the diagnosis?
A. CYSTITIS
B. SYMPTOMATIC BACTERIURIA
C. ASYMTOMATIC BACTERIURIA
D. PYELONEPHRITIS

27. A 32-year old G2P1 at 17 weeks gestation reports a history of Tetralogy of Fallot. She appears
comfortable on the exam table, however, she reports significant palpitations and dyspnea when
climbing the two flights of stairs to her apartment. Which New York Association class describes
her functional disability?
A. Class III
B. Class I
C. Class II
D. Class IV

28. For stillbirth evaluation,what should be the test of choice?


A. cytogenetic analysis (karyotyping)
B. chromosomal microarray analysis (CMA) (trans)
C. fluorescence in situ hybridization (FISH)
D. Whole genome sequencing (WGS) /Whole exome sequencing (WES)

29. Family Planning of choice given to lactating women is the following, except:
A. IUD
B. All of the choices
C. Implants
D. Combined Depo Injection
E. Progestin Only Pills

30. Janelle, G1P0 PU 25 weeks aog, teenage pregnancy feels insecure because of facial pustules
noted 1 week ago, her cheek looks so red, her OB said that it is related to coalescing sinuses.
What will you NOT advise Janel of her condition?
A. It usually complicates pregnancy
B. Topical or oral antimicrobials are primary treatment,
C. corticosteroids have also been used
D. Surgical drainage can be done

31. Karla, G0, is trying to conceive in the next few months, however, she have annoying skin
rashes, she came to your clinic asking for therapeutic agents to be avoided during pregnancy
except:
1 point
psoralen plus ultraviolet A
mycophenolate mofetil
biological agents
topical retinoids

32. Which of the following is NOT TRUE regarding the Intrinsic Pathway of activating the
coagulation system?
It is a faster process
Initially activates the Hageman factor
There is damage to vessel endothelium
Endotoxins are present

33. Which of the following medications is contraindicated in pregnant patients with asthma?
Oxytocin
Prostaglandin E1
None of the above
Prostaglandin E2
Prostaglandin F2A- guide book

34. TRUE ABOUT ANATOMIC RENAL CHANGES IN PREGNANCY.


there is vesicoureteral reflux due to satasis
there is increase in kidney size due to hypertrophy
all of the above- trans
there dilatation of the ureters and pelvis due to progesterone

35. GOALS of bronchial asthma management


Maintain normal lung function
Including long acting beta 2 agonists to control exacerbations
Decrease use of short acting beta 2 agonists- trans
Decrease exacerbations

36. The number of women who gets pregnant during an imperfect use of a FP method, is:
Pearl Index Rate
Typical Failure Rate - trans
Method Failure Rate
Theoretic Failure Rate

37. The Emotional Valor is seen at what part of the pregnancy?


Post-Partum Period
Third Trimester
Labor - trans
Second Trimester
38. Jean, a 17 y/o primigravid went to an obstetrician asking for elective termination of her
unwanted pregnancy at 6 weeks. The obstetrician refused to cooperate. What principle was
practiced?
1 point
autonomy
beneficence
non-malificence
justice

39. What is the preferred method of gastrointestinal tract evaluation during pregnancy?
1 point
Abdominal sonography
Computed tomography
Barium enema
Endoscopy - trans

40. All of the following except one are AHA Guideline in dealing with critically ill gravidas:
1 point
Establish intravenous access above the diaphragm
Relieve possible venacaval compression by left lateral displacement of the uterus
Administer 90% oxygen - williams
Assess hypotension which require therapy

41. What should be met before pregnancy in a post kidney transplant patient?
1 point
6 months post transplant with no recent or ongoing rejection
2 months post transplant and normotensive or with minimal antihypertensives
1 month with absent or minimal proteinuria
2 years post-transplant, with good general health and serum creatinine less than 2.0 mg/dL
(preferably <1.5 mg/dL - trans

42. What is the most common sex chromosome abnormality?


1 point
Klinefelter’s syndrome- williams page 265
47,XYY and 47,XXX
Cri-du-chat syndrome
Monosomy X syndrome

43. Post-Partum Depression can be prevented by:


1 point
Unplanned Pregnancy
Secured Environment
Stress
All of the choices
44. What is the most common causative agent in UTI during pregnancy?
1 point
Klebsiella
E. coli - williams page1052
Ureaplasma
Clostridium

45. Hypoxemia is defined as PaO2 less than:


1 point
60
80
90
70

46. Which is a characteristic disadvantage of a MULTIPHASIC Oral Contraceptive Pill?


1 point
Has varying amounts of Estrogen and Progestin
Difficulty in “Doubling Up
Decrease in breakthrough bleeding
Lesser Confusion

47. Progesterone Implants are administered via what route?


1 point
Intramuscular
Intra-cutaneous
Intradermal
Dermal
Subdermal

48. To compensate for the increased myocardial consumption, which antenatal change is most
important
1 point
Decreased anticoagulant activity
Increased thrombin levels
Increased blood volume
Decreased peripheral resistance

49. A 33 y/o multigravid consulted at the ER with physical findings of mild exophthalmos, sweaty
skin, and rales at the lung bases. Which of the following test will give you the most specific result
indicating the disease process?
1 point
ECG
Chest xray
Hematocrit
All of the choices
Thyroid function tests

50. What is the most common cause of Trisomy 21—Down Syndrome ?


1 point
isochromosome
robertsonian translocation
nondisjunction
mosaicism

51. A maternal adaptation that contributes to the hypercoagulable state:


1 point
.Increased RBC production
Increased plasma volume
Decreased peripheral vascular resistance
Decreased Protein C and S activity

52. What is the most common cause of heart failure in pregnancy?


1 point
Idiopathic
Severe preeclampsia
Chronic hypertension with superimposed preeclampsia
Chronic hypertension

53. One of the following can cause both cardiogenic and non cardiogenic pulmonary edema?
1 point
Acute hemorrhage
Vigorous intravenous fluid therapy
Hypertensive Cardiomyopathy
Pre eclampsia syndrome

54. WHAT IS THE MOST COMMON URINARY TRACT INFECTION?


1 point
CYSTITIS
ASYMTOMATIC BACTERIURIA
SYMPTOMATIC BACTERIURIA
PYELONEPHRITIS

55. Which anti Koch’s drug is not given during the maintenance phase
1 point
Rifampicin
Pyrazinamide
Isoniazid
Ethambutol

56. What is the most common inherited form of intellectual disability, autism or “autistic like”
behavior?
1 point
47XYY
47XXY
Fragile X Syndrome
Monosomy X

57. Important factor in precipitating cardiac failure


1 point
Blood loss
Superimposed preeclampsia
Septic shock
Hypertension

58. A 21-year-old female admitted for 3 day history of diarrhea with severe renal involvement and
no neurological symptoms was diagnosed with Thrombotic microangiopathy. What is the
cornerstone of treatment?
1 point
Plasma exchange
Hydration
IVIG
Dialysis

59. Ms Aila is a 35 year old nulligravid contemplating on elective freezing of her oocytes. if she
did pursue with this, what principle was practiced?
1 point
autonomy
beneficence
non-malificence
justice

60. Which of the following is NOT a Natural Family Planning Method?


1 point
Lactation Amenorrhea Method (LAM)
Coitus Interruptus
Billing’s Method
Rhythm Method
It should be NOTA but option is not given
61. JK, 35 year old newly wed and anxious of getting pregnant with congenital anomaly because
of her age, which of the following is not linked with her advance maternal age?
A. Patau’s syndrome
B. Down’s syndrome
C. Edward’s syndrome
D. Turner’s syndrome

62. What is the most common birth defect?


A. Metabolic defects
B. Cardiac Defects
C. Musculoskeletal defects
D. Neural-Tube Defects

63. Which of the following clinical signs defines progression from the warm phase of septic shock
to cold phase of septic shock?
A. Pulmonary Hypertension
B. Tachypnea
C. Oliguria
D. Leucocytosis

64. Helpful test for Cushing Syndrome


1 point
A. 24 hr urine collection for cathecolamines, metanephrines, or vanillylmandelic acid
B. Dexamethasone suppression test
C. Aldosterone-renin ratio
D. Elevated 24hr urine free cortisol excretion

65. Ysabel consulted at the ER for occasional hypogastric pain. Afraid that she is in labor, she
consulted and asked to be admitted. Due to the limited hospital capacity, Lisa was sent home
despite her plea to be admitted after she was examined. what principle was applied?
A. autonomy
B. beneficence
C. non-malificence
D. justice

66. This determines the way a pregnant woman reacts to the dynamic changes in pregnancy:
A. Psyche
B. Physical
C. Spirit
D. Soma

67. If structural abnormalities are identified, what is the first step to offer?
1 point
A. Whole genome sequencing (WGS) /Whole exome sequencing (WES)
B. chromosomal microarray analysis (CMA)
C. cytogenetic analysis (karyotyping)
D. fluorescence in situ hybridization (FISH)
68. Which of the following is compatible with life, but it is also the most common aneuploidy in
first-trimester losses?
1 point
A. Down’s syndrome
B. Patau’s syndrome
C. Turner’s syndrome
D. Edward’s syndrome

69. The time where there is love or resentment towards child for the feeling of being controlled:
A. Third Trimester
B. First Trimester
C. Second Trimester
D. Parturition

70. Which of the following is highly lethal?


A. Turner’s syndrome
B. Edward’s syndrome
C. Down’s syndrome
D. Patau’s syndrome

71. If a woman underwent thyroid ablation with therapeutic radioactive iodine, when is she allowed
to get pregnant?
A. After 6 months
B. After 6 weeks
C. After 3 months
D. After 3 weeks

72. What explains why some autosomal dominant diseases may appear to “skip” generations?
A. Penetrance
B. Advanced Paternal Age
C. Expressivity
D. Codominant Genes

73. CG, 40 years old first timer mother on her 30 weeks AOG is suspected to be carrying a fetus
with congenital anomalies and a noted hyperplacentosis.Other than this, there is a new onset of
hypertension. Which of the following is closely related to hypertension in pregnancy?
A. Turner’s syndrome
B. Patau’s syndrome
C. Down’s syndrome
D. Edward’s syndrome

74. Which tocolytic drug has the strongest association with the development of pulmonary edema
in obstetrical patients?
A. Terbutaline
B. Atosiban
C. Indomethacin
D. Magnesium sulfate

75. This condition is similar to a Male Pseudocyesis:


A. Hyperemesis Gravidarum
B. Factitious Pregnancy
C. Pica
D. Couvade

76. Which of the following best describes the remodeling of the heart that occurs in normal
pregnancy?
A. None of the choices
B. Concentric left ventricular mass expansion with spherical remodeling
C. Eccentric left ventricular mass expansion with longitudinal remodeling
D. Eccentric left ventricular mass expansion with spherical remodeling
E. Concentric left ventricular mass expansion with longitudinal remodeling

77. A 17 year old woman at 22 weeks gestation presents at the ER with a 3 day history of nausea,
vomiting and abdominal pain. The pain started in the middle of the abdomen and is now located
in her mid to upper right side. She is noted to have a temperature of 38C. she denies any history
of other medical and surgical problems. How does pregnancy alter the diagnosis and treatment
of the disease?
A. Owing to anatomical and physiologic changes in pregnancy, diagnosis is easier to make
B. The incidence is higher in pregnancy
C. The incidence is unchanged in pregnancy
D. Surgical treatment should be delayed since the patient is pregnant
E. Fetal outcome is improved with delayed diagnosis

78. A 29 year old in her 33 weeks gestation has been admitted in the emergency room because
of abdominal pain that has been increasing in severity for the past 24 hours. The pain is severe
and is radiating from the epigastrium to the back. The patient also complains of vomiting and has
not eaten or had bowel movement since the pain began. On examination, the patient is noted to
be ill looking lying on the bed with her knees drawn up. Her blood pressure is 100/70 mmhg, heart
rate of 110 beats per minute and temperature of 38.8C. On abdominal examination, the abdomen
is distended and tender in the epigastric area, with a fundic height of 31 cms above the symphysis.
Hypotonic bowel sounds are noted. Fetal heart rate is normal with no uterine contractions.
Ultrasound revealed a single fetus in vertex presentation, amniotic fluid is normal, placenta is
located in the anterior uterine wall and no previa is seen. Laboratory values showed mild
leukocytosis (WBC>12,000 cells/ml), hematocrit of 43%, mildly elevated SGOT, SGPT and
Bilirubin; and serum amylase of 180 U/dl. Urinalysis is normal. What is the most likely diagnosis?
A. Acute Pancreatitis
B. Severe pre eclamptic toxemia
C. Acute Cholecystitis
D. Acute Appendicitis
E. Acute degeneration of uterine myoma

79. LB, on her 34th week AOG, was admitted for very high BP - 220/120mm Hg. if her BP remains
uncontrolled, she was told that she needs to undergo emergency CS despite having a premature
baby. if LB chose not to undergo the cesarean section, what principle was applied?
A. autonomy
B. beneficence
C. non-maleficence
D. justice

80. What is the main hormone responsible for hyperemesis gravidarum?


A. estrogen
B. hCG
C. relaxin
D. progesterone

81 Which of the following is NOT a beneficial effect of COC?


1 point
Decreased Risk of Endometrial Ca
Decrease epithelial ovarian cancer
Improves Bone mineral density
Prevents Cervical Cancer
Lowers PID Incidence

82 Which of the following is a NON-HORMONAL Family Planning Method?


1 point
Implants
Oral Contraceptive Pills
Dermal Patch
Mirena IUD
Copper T380 IUD

83 WHAT IS THE MOST COMMON FETAL CONGENITAL ANOMALY?


1 point
G6PD deficiency
neural tube defects
heart defects
Down syndrome

84 When should treatment of asymptomatic bacteriuria be given? A clean-voided specimen


containing__________
1 point
only 20,000 organisms/mL
only 50,000 organisms/mL
more than 100,000 organisms/mL
any of the choices

85 Which of the following presents as molar pregnancy?


1 point
All of the choices
Tetraploid
Digynic triploid
Diandric triploid

86 This type of emotion is involved during Quickening:


1 point
Fantasy
Fear
Valor
Acceptance

87 MOst common cause of thyrotoxicocis in pregnancy


1 point
Thyroiditis
Graves disease
Hashimoto’s disease
GTD

88 Jaime, a mukbang vlogger, G2P1 (1001) PU


31 weeks aog came in due to severe pruritus after eating 5 trays of chicharong bulaklak, she also
loves to eat in sangyupsal restaurants. Upon work up, noted elevated serum bile acid levels, and
hepatic aminotransferase, what is the pregnancy specific dermatosis of Jayme?
1 point
Intra-hepatic cholestasis of pregnancy
Pruritic urticarial papules and plaques of pregnancy (PUPPP)
Atopic eruption of pregnancy (AEP)
Pemphigoid gestationis

89 What is the most easily recognized numerical chromosomal abnormalities?


1 point
Triploidy
Monosomy
Polyploidy
Aneuploidy

90 True about Standard Nomenclature in Genetics EXCEPT:


1 point
chromosomes correspond to the number of centromeres
description of any structural variation is listed also but not the specific abnormalities
none of the choices
total number of chromosomes is listed first followed by the sex chromosomes

91 During abnormal platelet activation, which of


the following is decreased?
1 point
Prothrombin
Thrombin
Fibrinogen
Prostacyclin

92 The increased neonatal death rate seen in gravidas who undergo non obstetrical surgery is
attributable to higher rates of which of the following?
1 point
Congenital anomalies in patients with 1st trimester procedures
Preterm birth
Fetal metabolic acidemia associated with emergency cesarean delivery
Fetal growth restriction

93 Which of the following detects numerical abnormalities (aneuploidy)?


1 point
Whole genome sequencing (WGS) /Whole exome sequencing (WES)
cytogenetic analysis (karyotyping)
chromosomal microarray analysis (CMA)
fluorescence in situ hybridization (FISH)

94 Katie, a 19 y/o student found out she was 7 weeks pregnant. Fearful of what her parents will
say, she took pills to induce abortion. What principle was violated?
1 point
Autonomy
Beneficence
Non-Malificence
Justice
All of the above

95 Pica persists of at least how many weeks?


1 point
2 weeks
4 weeks
1 weeks
3 weeks
96 A type of somatoform disorder is:
1 point
Couvade
Pica
Option 6
Pseudocyesis
Depression

97 Which of the following is the most dreaded reason why UTI must be monitored and treated
during pregnancy?
1 point
Bladder sensations can also be obscured
urinary stasis, vesicoureteral reflux, and diabetes is increased during pregnancy
MAY LEAD TO UROSEPSIS
THERE IS REVERSAL OF T-helper cell—Th1/Th2 ratio

98 TRUE of PTU compared to methimazole


1 point
There is twofold increase in major fetal malformation
Best given during second trimester
Has less incidence of transient leukopenia
Has higher hepatotoxicity compared to methimazole

99 Anjie, a 24 y/o G2P0 (0010) went to the emergency room for abdominal pain. On history, it
was revealed that her first pregnancy was also an ectopic pregnancy where her fallopian tube
was removed. Thinking of possible sterility if she undergoes another operation, she refuses to be
admitted. What principle is shown here?
1 point
Autonomy
Beneficence
Non-malificence
Justice
All of the above

100 Which is NOT transferable or does NOT cross the placenta


1 point
TSH
T3
All crosses the placenta
T4

You might also like